Download APEA TEST BANK-with 100% verified answers-2024-2025 docx and more Exams Nursing in PDF only on Docsity! APEA TEST BANK-with 100% verified answers-2024-2025 South University A patient with iron deficiency anemia takes iron supplementation daily. What should he be advised to avoid within a couple of hours of taking iron? An antacid Leukemia may have varied clinical presentations. Which characteristic would be unusual to find in a patient with leukemia? Sickle shaped cells A 66 year-old African American male complains of pain in his trunk, especially his ribs. Cardiovascular disease is ruled out. He has a normocytic, normochromic anemia with hypercalcemia. The differential diagnosis should include: multiple myeloma. What hallmark finding is associated with both B12 and folate deficiencies? Macrocytosis A patient demonstrates leukocytosis. This means: he has an infection of unknown origin. What statement is true about anemia in older adults? Anemia may have more than one origin and co-exist in older adults. A patient has been treated for HIV infection with anti-retroviral therapy. He is stable. How often should CD4 counts be repeated? Every 3-6 months A patient presents with hematuria, RBC casts, and proteinuria. What is a likely explanation? Glomerulonephritis A patient demonstrates leukocytosis. This means: he has an infection of unknown origin. A patient is having an allergic reaction to seafood. Which white cell will probably be increased? Eosinophils A female patient has been diagnosed with Glucose-6-phosphate dehydrogenase deficiency (G6PD). What should be done to prevent lysis of red cells in this patient? Avoid aspirin and sulfa drugs A 75 year-old patient who has multiple chronic diseases has been in very poor health for a decade. What type of anemias is he most likely to exhibit? Folic acid and pernicious A patient has been diagnosed with HIV. The patient’s viral load was ordered. What other test may be ordered to assess the status of the patient’s immune system? CD4 cell count A 70 year-old male has lymph nodes in his axillary and inguinal areas that are palpable but non-tender. He states that he feels well today. What should be included in a Lymphoma APEA TEST BANK-with 100% verified answers-2024-2025 South University differential diagnosis for this patient? A two year-old with sickle cell anemia (SCA) should receive which immunizations? All routine childhood immunizations at the usual time A three year-old child presents with hematuria, petechiae, and a platelet count of 50,000 (Normal = 150,000- 450,000/ml). The rest of his CBC is normal. He had an upper respiratory infection about 2 weeks ago. On exam today, he is found to have petechiae and bruises. The most likely diagnosis is: idiopathic thrombocytopenia purpura (ITP). A child and father live in an old house. They both are found to be lead toxic. What type anemia is typically observed in patients who are lead toxic? Iron deficiency anemia A patient with diarrhea has a stool specimen positive for WBCs. What does this indicate? A viral infection A 26 year-old female has thalassemia minor. What should be limited in her diet to avoid hepatotoxicity? Multi-vitamin with iron A patient is found to have eosinophilia. An expected finding is: asthma exacerbation. An African American male complains of pain in his back and trunk. He is diagnosed with multiple myeloma. He is probably: about 65 years old. . A patient has heavy menses. Which lab value below reflects an iron deficiency anemia? Increased TIBC An obese 78 year-old male with poorly controlled hypertension and diabetes has a normocytic, normochromic anemia. This anemia is likely: associated with chronic disease. . An elderly male diagnosed with a microcytic, hypochromic anemia: may have a GI bleed. Which suggestion below is the standard for Iron supplementation in treating iron deficiency anemia in infants divided doses between meals and children? with orange juice A patient with pernicious anemia may be observed to have: glossitis. APEA TEST BANK-with 100% verified answers-2024-2025 South University A patient has CBC results that indicate a microcytic, hypochromic anemia. The nurse practitioner should suspect: iron deficiency anemia. A 12 month-old was screened for iron APEA TEST BANK-with 100% verified answers-2024-2025 South University p r ti A patient is found to have an anemia. The patient’s MCV is normal. The patient’s normocytic. anemia can be described as: A B-12 deficiency can produce: pernicious anemia. Thalassemia minor can be recognized by: microcytic, hypochromic red cells. A patient had a splenectomy after an automobile accident 3 months ago. Patients who are asplenic are: An example of a first generation cephalosporin used to treat a skin cephalexin. infection is: A 4 year-old has been diagnosed found on the with measles. The nurse inside of the practitioner identifies Koplik’s cheek and are more spots. These are: granular. susce The main difference between cellulitis layer of skin tible to and erysipelas is the: involvement. bacte A patient reports that he found a ial tick on himself about one month infec ago. He reports that there is a red on. circle and a white center near where he remembers the tick bite. ELISAHe did not seek treatment at the time. Today he complains of myalgias and arthralgias. What laboratory test can be used to help diagnose Lyme disease? A patient who is at high risk for examine his skin cancer should: skin monthly for changes. A patient has a “herald patch” and is diagnosed with pityriasis rosea. On the chest Where is the “herald patch” found? Which of the following antibiotics Fluoroquinolones APEA TEST BANK-with 100% verified answers-2024-2025 South University may increase the likelihood of photosensitivity? A 60 year-old patient is noted to have rounding of the distal Hepatic cirrhosisphalanx of the fingers. What might have caused this? A patient exhibits petechiae on both lower legs but has no other Order a CBC complaints. How should the NP proceed? When can a child with chickenpox After all lesions have crusted over return to daycare? Hand-foot-and-mouth disease and are viral infections caused by herpangina: Coxsackie viruses. What is the proper technique to safely remove a tick from a Pull it off with tweezers human? Most cases of atopic dermatitis topical steroids. exacerbation are treated with: The primary therapeutic intervention for patients who anti-histamines. present with hives is: A patient is diagnosed with tinea pedis. A microscopic examination of the sample taken from the hyphae. infected area would likely demonstrate: A patient has been diagnosed with scabies. What is the medication of Permethrin choice to treat this? An adolescent has acne. The nurse practitioner prescribed a benzoyl peroxide product for him. What Photosensitivity of the skin can important teaching point should occur be given to this adolescent regarding the benzoyl peroxide? A patient presents with plaques on the extensor surface of the elbows, knees, and back. The plaques are plaque psoriasis. erythematous and there are thick, APEA TEST BANK-with 100% verified answers-2024-2025 South University well marginated half moon macules on the inner thigh. Doxycycline A skin lesion which is a solid mass papule. APEA TEST BANK-with 100% verified answers-2024-2025 South University is described as a: Impetigo is characterized by: honey-colored crusts. A 10 year-old has thick, demarcated plaques on her elbows. Which features are suggestive of psoriasis? The agent commonly used to treat patients with scabies is permethrin. How often is it applied to eradicate scabies? Which test is NOT suitable to diagnose shingles if the clinical presentation is questionable? A 74 year-old is diagnosed with Silvery scales that are not pruritic Once Complete blood count (CBC) shingles. The NP is deciding how to best manage her care. What should be prescribed? The term caput succedaneum refers to: A pregnant mother in her first An oral antiviral agent scalp edema. trimester has a 5 year-old who has There is a risk of fetal death if she Fifth Disease. What implication does this have for the mother? A topical treatment for basal cell carcinoma is: The most common form of skin cancer is: A skin disorder has a hallmark finding of silvery scales. What word below describes this common condition? The most common place for basal cell carcinoma to be found is the: A low potency topical hydrocortisone cream would be most appropriate in a patient who has been diagnosed with: A patient with a positive history of a tick bite about 2 weeks ago and erythema migrans has a positive ELISA for Borrelia. The Western blot is positive. How should he be managed? APEA TEST BANK-with 100% verified answers-2024-2025 South University becomes infected. 5-fluorouracil. basal cell carcinoma. Chronic face. atopic dermatitis. He should receive doxycycline for Lyme disease. APEA TEST BANK-with 100% verified answers-2024-2025 South University A patient is found to have koilonychia. What laboratory test would be prudent to perform? Which of the following areas of the body has the greatest percutaneous absorption? A patient calls your office. He states that he just came in from the woods and discovered a tick on his upper arm. He states that he has removed the tick and the area is slightly red. What should he be advised? A patient will be taking oral terbinafine for fingernail fungus. The NP knows that: An elderly patient has been diagnosed with shingles on the right lateral aspect of her trunk. It appeared initially yesterday. It is very painful. How should she be managed? A young child has developed a circumferential lesion on her inner forearm. It is slightly raised, red and is pruritic. It is about 2.5 cm in diameter. This is probably related to: Which chronic skin disorder primarily affects hairy areas of the body? A microscopic examination of the sample taken from a skin lesion indicates hyphae. What type infection might this indicate? A 40 year-old female patient presents to the clinic with multiple, painful reddened nodules on the anterior surface of both legs. She is concerned. These are probably associated with her history of: APEA TEST BANK-with 100% verified answers-2024-2025 South University Complete blood count Genitalia No treatment is needed. Terbinafine is a potent inhibitor of the CYP 3A4 enzymes. An oral antiviral agent and pain medication. the child’s new cat. Seborrheic dermatitis Fungal ulcerative colitis. What advice should be given to a A parent may experience joint APEA TEST BANK-with 100% verified answers-2024-2025 South University parent who has a child with Fifth Disease? What finding characterizes shingles? A 70 year-old is diagnosed with multiple cherry angiomas. The nurse practitioner knows that: A patient with diabetes has a right lower leg that has recently become edematous, erythematous, and tender to touch over the anterior shin. There is no evidence of pus, but the leg is warm to touch. What is the most likely diagnosis to consider? The American Cancer Society uses an ABCDE pneumonic to help patients develop awareness of suspicious skin lesions. What does the “B” represent? A 9 year-old has been diagnosed with chickenpox. A drug that should be avoided in him is: Mr. Johnson is a 74 year old who presents with a pearly-domed nodular looking lesion on the back aches and pains. Unilateral dermatomal rash these may bleed profusely if ruptured. Cellulitis Border aspirin. Basal cell carcinoma of the neck. It does not hurt or itch. What is a likely etiology? A 3 year-old female had a fever of 102 degrees F for 3 days. Today she woke up from a nap and is afebrile. She has a maculopapular rash. Which statement is true? The lesions seen in a patient with folliculitis might be filled with: A child has 8-10 medium brown café au lait spots > 1 cm in diameter. The differential diagnosis should include: A patient presents with small vesicles on the lateral edges of his fingers and intense itching. On close inspection, there are small APEA TEST BANK-with 100% verified answers-2024-2025 South University examine his skin monthly for changes. Tachyphylaxis basal cell carcinoma. Pityriasis rosea indicative of candidal infection. Encourage the caregiver to change the infant’s head position Amoxicillin strept infection. Call the dermatologist to report the elevated BP Which of the following skin lesions Actinic keratosis APEA TEST BANK-with 100% verified answers-2024-2025 South University Question: A 40 year-old female patient presents to the clinic with multiple, painful reddened nodules on the anterior surface of both legs. She is concerned. These are probably associated with her history of: deep vein thrombosis.phlebitis. Incorrectulcerative colitis. Correctalcoholism. Explanation: These nodules describe erythema nodosum. These are most common in women aged 15-40 years old. They are typically found in pretibial locations and can be associated with infectious agents, drugs, or systemic inflammatory disease like ulcerative colitis. They probably occur as a result of a delayed hypersensitivity reaction to antigens. It is not unusual to find polyarthralgia, fever, and or malaise that precede or accompany the skin nodules. Question: A patient reports that he found a tick on himself about one month ago. He reports that there is a red circle and a white center near where he remembers the tick bite. He did not seek treatment at the time. Today he complains of myalgias and arthralgias. What laboratory test can be used to help diagnose Lyme disease? CBCLyme titerELISA CorrectSkin scraping Explanation: A detailed history should always precede testing for Lyme Disease. The red circle with the white center is likely erythema migrans (EM). EM is the characteristic skin lesion of Lyme Disease (and other illnesses) and usually occurs within one month following the tick bite. Many learned authorities including the Infectious Diseases Society of America conclude that individuals should not be screened/tested for Lyme disease unless they have a high probability of having Lyme disease. In this case, historical features coupled in the elderly is a premalignant condition? A patient has suspected scarlet fever. He likely has a sandpaper rash and: a positive rapid strept test. A 6 year-old patient with sore throat has coryza, hoarseness, and Viral etiology diarrhea. What is the likely etiology? A 74 year-old male patient has sustained a laceration to his foot. His last tetanus shot was moreTetanus, diphtheria, and acellular than 10 years ago. He haspertussis (Tdap) completed the primary series. What should be recommended? The best way to evaluate jaundice associated with liver disease is to the sclera, skin, and lips. observe: APEA TEST BANK-with 100% verified answers-2024-2025 South University with physical exam support the diagnosis, and thus screening. The most common initial serologic test for screening is an ELISA. If it is positive, it should be confirmed with a Western blot. Unfortunately, there are a large number of false positives and so a confirmation should be performed Question: An adolescent takes isotretinoin for nodulocystic acne. She is on oral contraceptives. Both were prescribed by the dermatologist. The adolescent arrives in your clinic with a sinus infection. Her temperature is 99.5 degrees F and her blood pressure is 160/100. How should this be managed? Call the dermatologist to report the elevated BP CorrectTreat the sinus infection and recheck the BP in one week IncorrectDiscontinue the isotretinoin todayDiscontinue the oral contraceptive today Explanation: The nurse practitioner is responsible for treating the sinus infection but has also become aware of a potentially harmful situation involving the elevated blood pressure and oral contraceptive use. The safest and most professional action is to call the dermatologist to discuss your concerns regarding the elevated BP and concomitant oral contraceptive use since this potentially increases the risk of stroke in this adolescent. Care and professional courtesy should be exercised when discontinuing a medication that another provider has initiated. Professional courtesy is extended to the prescriber by calling them prior to discontinuing a medication they have ordered. Question: The agent commonly used to treat patients with scabies is permethrin. How often is it applied to eradicate scabies? Once CorrectOnce daily for 3 daysTwice daily for 3 daysOnce daily for one week Incorrect Explanation: A single whole body application of permethrin is usually successful in eradicating infection with scabies. It is applied over the entire body from the neck down. The lotion is left on and then showered off 8-12 hours later. All contacts must be treated at the same time and all potential fomites (bed linen, mattresses, cloth furniture, etc.) must be treated as well. Permethrin can be sprayed on cloth fomites or they can be bagged for several days, washed and dried in washing machine and dryer. Ironing clothes after washing them is acceptable. Question: An infant is diagnosed with diaper dermatitis. Satellite lesions are visible. This should be treated with a: moisture barrier like zinc oxide.topical anti-fungal agent. Correcttopical anti-bacterial agent.low potency steroid cream. Explanation: The finding of satellite lesions associated with diaper dermatitis indicates a Candidal infection. This patient will be most effectively treated with a topical anti-fungal agent, allowing the lesions to be exposed to air for periods of time (like during a nap). A moisture barrier like zinc oxide is more beneficial when the diaper dermatitis is due to irritants like prolonged exposure to urine or feces. A low potency steroid cream should be used with caution in an infant with a fungal infection. A low potency cream in conjunction with an anti- fungal can be helpful if there is a great amount of underlying inflammation but has the potential to worsen the infection. Question: A 16 year-old has been diagnosed with Lyme disease. Which drug should be used to treat him? Doxycycline CorrectAmoxicillin-clavulanateTrimethoprim-sulfamethoxazoleCephalexin Explanation: Doxycycline is frequently chosen first line to treat Lyme Disease. However, numerous studies have demonstrated that amoxicillin and cefuroxime have equal efficacy as doxycycline in treatment of early Lyme Disease. These drugs are recommended in patients who exhibit erythema migrans. Doxycycline is not recommended in children less than 9 years of age Question: APEA TEST BANK-with 100% verified answers-2024-2025 South University Most oral antifungal agents inhibit the 3A4 enzymes in the cytochrome P450 system. This is why they must be used with extreme caution (or not used) in patients who consume medications that need 3A4 enzymes for metabolism. And, liver enzymes must be monitored in patients who take oral antifungal medications and discontinued if elevations are >2.5 times the upper limits of normal. There is no oral agent that has a 95% cure rate for fingernail fungus (tinea unguium). This can be a difficult infection to clear even if oral antifungal agents are utilized. A topical antifungal agent typically will not clear the infection if the nail matrix is involved. There is anecdotal evidence that menthol ointments or bleach may cause resolution when used topically. Generally, resolution of fingernail fungus occurs more rapidly than toenail fungus because toenails grow at slower rates than fingernails. Question: Which of the following areas of the body has the greatest percutaneous absorption? Sole of the footScalp IncorrectForeheadGenitalia Correct Explanation: Genitalia have the highest percutaneous absorption across the entire body. This is important because low potency creams will act with greater potency in this area. Always start with low potency creams in the genitalia. The sole of the foot has the lowest percutaneous absorption followed by the scalp, forehead, and genitalia. Therefore, the sole of the foot will require more potent vehicles to enhance absorption. Question: A patient who has been in the sun for the past few weeks is very tanned. He has numerous 3-6 mm light colored flat lesions on his trunk. What is the likely etiology? Tinea corporisTinea unguiumTinea versicolor CorrectHuman papilloma virus Explanation: Tinea versicolor is typically visualized during the spring and summer months when a patient has become tanned. The areas that are infected do not tan and so become very noticeable. The chest and back are common areas to observe tinea versicolor. There can be 100 or more in some infections. This can be treated with topical selenium sulfide or an oral antifungal agent. Question: The nurse practitioner is examining a 3-month old infant who has normal development. She has identified an alopecic area at the occiput. What should be done? Order a TSHOrder a hydrocortisone creamSuspect child abuseEncourage the caregiver to change the infant’s head position Correct Explanation: In a normally developing infant, an alopecic area at the occipital area is generally because the infant has been placed in the supine position during sleeping and waking hours. Prolonged pressure on the occipital area can restrict hair growth. This is normal and will resolve when the infant begins to have better head control and movement; and he begins to have less pressure on the occipital area. Sometimes coarse, dry hair can be indicative of hypothyroidism; not necessarily alopec Question: A patient calls your office. He states that he just came in from the woods and discovered a tick on his upper arm. He states that he has removed the tick and the area is slightly red. What should he be advised? No treatment is needed. CorrectHe should be prescribed doxycycline. IncorrectHe needs a topical scrub to prevent Lyme Disease.He should come to the office for a ceftriaxone injection. Explanation: Many factors must be present for a patient to develop Lyme Disease from a tick bite. First, the tick must belong to Ixodes species. The tick must have been attached for at least 48-72 hours before disease can be spread. Time of year, stage of organism's development, and others all affect transmission. There is no need for prophylactic treatment in this case because the tick has not been present long enough, though, many patients will feel antibiotics are necessary. APEA TEST BANK-with 100% verified answers-2024-2025 South University Question: An example of a first generation cephalosporin used to treat a skin infection is: cephalexin. Correctcefuroxime.cefdinir.ceflamore. Explanation: Two common first generation cephalosporins used to treat skin and skin structure infections are cephalexin and cefadroxil. These are taken 2-4 times daily and are generally well tolerated. These antibiotics provide coverage against Staphylococcus and Streptococcus, common skin pathogens Question: Which of the following skin lesions in the elderly is a premalignant condition? XanthelasmaChalazionHordeolumActinic keratosis Correct Explanation: Actinic keratosis is a premalignant condition of the skin and is considered an evolving carcinoma in situ. It is a precursor of squamous cell carcinoma. The lesions are usually multiple in occurrence and sit on an erythematous base. They appear dry, scaly, and flat and are usually secondary to sun damaged skin so can be found on sun exposed areas. The most common sites are the face, ears, lateral forearms, and tops of hands. Question: A 74 year-old is diagnosed with shingles. The NP is deciding how to best manage her care. What should be prescribed? An oral antiviral agent CorrectAn oral antiviral agent plus an oral steroidAn oral antiviral agent plus a topical steroidA topical steroid only Explanation: An oral antiviral agent such as acyclovir, famciclovir, valacyclovir should be prescribed, especially if it can be initiated within 72 hours after the onset of symptoms. The addition of oral corticosteroids to oral antiviral therapy demonstrates only modest benefit. Adverse events to therapy are more commonly reported in patients receiving oral corticosteroids. There is no evidence that corticosteroid therapy decreased the incidence or duration of post-herpetic neuralgia or improved quality of life. Corticosteroids should be limited to use in patients with acute neuritis who have not derived benefit from opioid analgesics. Question: A child with a sandpaper textured rash probably has: rubeola.strept infection. Correctvaricella.roseola. Incorrect Explanation: Streptococcal infections can present as a sandpaper textured rash that initially is felt on the trunk. Rubeola, measles, produces a blanching erythematous “brick-red” maculopapular rash that begins on the back of the neck and spreads around the trunk and then extremities. Varicella infection produces the classic crops of eruptions on the trunk that spread to the face. The rash is maculopapular initially and then crusts. Roseola produces a generalized maculopapular rash preceded by 3 days of high fever. Question: Patients with atopic dermatitis are likely to exhibit: Itching. Correctasthma and allergic bronchitis.nasal polyps and asthma.allergic conjunctivitis and wheezing. Explanation: APEA TEST BANK-with 100% verified answers-2024-2025 South University Atopic dermatitis is diagnosed on clinical presentation and includes evidence of pruritic skin. It is recurrent and often begins in childhood. For decades the "atopic triad" has been used to refer to patients with atopic APEA TEST BANK-with 100% verified answers-2024-2025 South University usually found in a symmetrical distribution on the scalp, elbows, knees, and/or back. The size of the lesions ranges from 1-10 cm in diameter. Usually the plaques are asymptomatic, but may be mildly pruritic. Scaly lesions found on the scalp are not specific to psoriasis and could be seborrheic dermatitis. A scaly border around the plaque could describe the lesions associated with pityriasis rosea. Question: What finding characterizes shingles? Pain, burning, and itchingUnilateral dermatomal rash CorrectGrouped vesiclesResolution of rash and crusting Explanation: Shingles is herpes zoster. It characteristically affects a single dermatome. Grouped vesicles on an erythematous base can be seen in some patients with shingles, but this is not unique to shingles. In fact, it is typical in many viral infections. Crusting may be seen with shingles, chicken pox, or impetigo. Pain, burning, and itching describes the symptoms that some patients have with shingles, but not all patients report itching with shingles. Question: A patient presents with small vesicles on the lateral edges of his fingers and intense itching. On close inspection, there are small vesicles on the palmar surface of the hand. What is this called? Seborrheic dermatitisDyshidrotic dermatitis CorrectHerpes zosterVaricella zoster Explanation: This dermatitis is intensely pruritic and involves the palms and soles and lateral aspects of the fingers. Over a couple of weeks, the vesicles desquamate. Recurrences are common. Seborrheic dermatitis affects only hairy areas of the body. The vesicles might raise suspicion of a viral infection, but this is not the case. Question: A patient exhibits petechiae on both lower legs but has no other complaints. How should the NP proceed? Refer to hematologyOrder a CBC CorrectOrder blood culturesStop aspirin and re-assess in one week Explanation: The presence of petechiae on the lower legs (or anywhere on the body) should prompt the NP to consider a problem that is platelet related. A CBC should be checked to assess the platelet count and any evidence of anemia from blood loss. If the platelet count is found to be low, referral to hematology should be done. Blood cultures are of no value in this patient who has no other complaints. Question: A 16 year-old male has nodulocystic acne. What might have the greatest positive impact in managing his acne? Retin-A® plus minocyclineBenzoyl peroxide plus erythromycinIsotretinoin (Accutane®) CorrectOral antibiotics Explanation: Nodulocystic acne is the most severe form of acne vulgaris. Nodules and cysts characterize this disease. They can be palpated and usually seen on the skin, although, they actually are under the skin’s surface. They develop when the follicle wall ruptures and leaks pus and cell contents in the dermis. The contaminated material infects adjoining follicles and the nodule develops. Isotretinoin is the only known effective treatment. Question: A patient with a positive history of a tick bite about 2 weeks ago and erythema migrans has a positive ELISA for Borrelia. The Western blot is positive. How should he be managed? APEA TEST BANK-with 100% verified answers-2024-2025 South University He should receive doxycycline for Lyme disease. CorrectHe should receive penicillin for Rocky Mountain spotted fever (RMSF).He does not have Lyme disease or RMSF.He needs additional testing to confirm Lyme disease. Explanation: The first serologic test for Lyme disease is the ELISA. If this is positive, it should be confirmed. In this case, it was confirmed by a Western blot and it is positive. This patient can be diagnosed with Lyme disease. The appropriate treatment for treatment of erythema migrans is doxycycline, amoxicillin, or cefuroxime for 21 days. All three medications were found to be of equal efficacy. Question: A pregnant mother in her first trimester has a 5 year-old who has Fifth Disease. What implication does this have for the mother? She does not have to worry about transmission to the fetus.She may get a mild case of Fifth disease.There is a risk of fetal death if she becomes infected. CorrectThe mother should have a fetal ultrasound today. Explanation: Pregnant mother should avoid exposure to patients with known Fifth disease. However, the risk of transmission is very low. She should avoid exposure to aplastic patients who are infected because they are highly contagious. Infection during pregnancy is associated with 10% fetal death. There is no need for an ultrasound today. This pregnant patient does not have evidence of disease. She should be monitored for a rash which could indicate infection. Question: A patient is diagnosed with tinea pedis. A microscopic examination of the sample taken from the infected area would likely demonstrate: hyphae. Correctyeasts.rods or cocci.a combination of hyphae and spores. Explanation: Under microscopic exam, hyphae are long, thin and branching, and indicate dermatophytic infections. Hyphae are typical in tinea pedis, tinea cruris, and tinea corporis. Yeasts are usually seen in candidal infections. Cocci and rods are specific to bacterial infections. Question: A topical treatment for basal cell carcinoma is: sulfacetamide lotion.5-fluorouracil. Correcttetracycline lotion.trichloroacetic acid. Incorrect Explanation: Several treatments exist for basal and squamous cell carcinoma. The majority are simple procedures like cryotherapy, electrodessication, surgical excision, and a topical treatment like 5-fluorouracil (5-FU). The other agents listed are not used to treat basal or squamous cell carcinoma. 5-FU works by inhibiting DNA synthesis. It is effective if used for superficial basal cell carcinomas. It is available in cream and solution and is usually applied twice daily for 3-6 weeks. Question: The best way to evaluate jaundice associated with liver disease is to observe: blanching of the hands, feet, and nails.the sclera, skin, and lips. Correctthe lips, oral mucosa, and tongue.tympanic membrane and skin only. Explanation: Looking at the sclera allows the examiner to see jaundice most easily and reliably. Jaundice may also appear in the palpebral conjunctiva, lips, hard palate, undersurface of the tongue, tympanic membrane, and skin. Jaundice in adults is a result of liver disease usually, but can be due to excessive hemolysis of red blood cells. In infants the usual cause is hemolysis of red blood cells as is seen in physiologic jaundice. APEA TEST BANK-with 100% verified answers-2024-2025 South University Question: A patient has been diagnosed with MRSA. She is sulfa allergic. Which medication could be used to treat her? AugmentinTrimethoprim-sulfamethoxazole (TMPS)CeftriaxoneDoxycycline Correct Explanation: MRSA is methicillin resistant Staph aureus. This is very common in the community and is typically treated with sulfa medications like TMPS (Bactrim DS and Septra DS). If the patient is sulfa allergic, this could not be used. A narrow spectrum antibiotic that can be used is doxycycline or minocycline. It is given twice daily and is generally well tolerated. MRSA is resistant to the antibiotics mentioned and so they should NOT be used to treat it. Question: An adolescent has acne. The nurse practitioner prescribed a benzoyl peroxide product for him. What important teaching point should be given to this adolescent regarding the benzoyl peroxide? Don’t apply this product more than once dailyThis often causes peeling of the skinPhotosensitivity of the skin can occur CorrectHypersensitivity can occur with repeated use Explanation: Benzoyl peroxide can produce sensitivity to the sun and so adolescents should be informed of this. This product can be used twice daily. It can cause peeling of the skin, but this is not a frequent occurrence. Hypersensitivity can occur with any topical product and is not specific to benzoyl peroxide. Question: Which of the following lesions never blanches when pressure is applied? Spider angiomaSpider veinPurpura or petechiae CorrectCherry angioma Explanation: Blanching with pressure over spider angiomas always occurs. Spider veins and cherry angiomas usually blanch with pressure. Purpura and petechiae never blanch with pressure. Purpura and petechiae represent an extravasation of blood under the skin. This will not blanch. This is usually observed in patients with thrombocytopenia or trauma. Question: A patient with a primary case of scabies was probably infected: 1-3 days ago.1 week ago.2 weeks ago.3-4 weeks ago. Correct Explanation: The incubation period for scabies is about 3-4 weeks after primary infection. Patients with subsequent infections with scabies will develop symptoms in 1-3 days. The classic symptom is itching that is worse at night coupled with a rash that appears in new areas over time. Question: The lesions seen in a patient with folliculitis might be filled with: blood.pus. Correctfluid.serous fluid. Incorrect Explanation: Folliculitis is a superficial inflammation of hair follicles usually caused by bacteria. As a superficial infection, it involves only the epidermis. When this occurs, there are usually numerous pustular lesions. The composition of pus is dead white cells and other cellular debris. Question: The term caput succedaneum refers to: cradle cap. Incorrectatopic dermatitis.scalp edema. Correctasymmetric head shape. APEA TEST BANK-with 100% verified answers-2024-2025 South University Explanation: APEA TEST BANK-with 100% verified answers-2024-2025 South University Skin cancers are divided into two major groups: non-melanoma and melanoma skin cancer. Basal cell carcinoma is considered non-melanoma skin cancer and is the most common form of skin cancer in the US. It is most common in 40-60 year-olds, but can be found in any age if the skin is regularly exposed to sunlight or ultraviolet radiation. Basal cell carcinoma grows slowly and if not treated can spread to surrounding areas of tissue or bone. Question: The nurse practitioner identifies satellite lesions in a 6 month-old infant. These are: worrisome in an infant this age.indicative of candidal infection. Correctfound in the axilla bilaterally.completely benign lesions present at birth. Explanation: Satellite lesions are common in the diaper area of infants with a candidal diaper infection. They generally are associated with a beefy red rash. The skin creases are more commonly affected, but satellite lesions can be found anywhere that Candida can be found.\ Question: A patient has seborrheic dermatitis. Which vehicle would be most appropriate to use in the hairline area to treat this? SolutionCreamPowderFoam Correct Explanation: Seborrheic dermatitis affects the hairy areas of the body. In hairy areas of the body, foams are specifically used because they spread easily and are transparent. Lotions can be used in hairy areas because they provide a cooling, drying effect and are transparent. Lotions are the weakest of all vehicles. Creams and powders could be used, but patients prefer other vehicles and so compliance is less with these. Question: An example of a premalignant lesion that develops on sun-damaged skin is: actinic keratosis. Correctbasal cell carcinoid. Incorrectsquamous cell carcinoma.molluscum contagiosum. Explanation: Actinic keratoses (AK) are a result of solar damage to the skin. They are most common on the face, bald scalp, and forearms. Patients who present with AK usually have multiple of them. A characteristic that helps identification of AK is an area of erythema that surrounds the lesion. AK is better felt than seen. Question: A 9 year-old female has presented to your clinic because of a rash on the left, upper area of her anterior trunk. She is embarrassed and very reticent to lift her blouse because her nipple will be exposed. How should the NP proceed? Examine the patient with the area covered as much as possible IncorrectThe NP should lift the blouse to expose the area for examAsk the accompanying caregiver to expose the area for examExamine all other areas of the trunk, then ask the child to lift her blouse Correct Explanation: The NP must visualize the area during exam so that an appropriate diagnosis can be made. Since the child is 9 years old, she should be given the opportunity to remove her blouse before allowing another adult to do this. Allowing the child to do this will put “control” in her hands. Question: A 6 year-old patient with sore throat has coryza, hoarseness, and diarrhea. What is the likely etiology? Group A Streptococcus IncorrectH. parainfluenzaeViral etiology CorrectMycoplasma APEA TEST BANK-with 100% verified answers-2024-2025 South University Explanation: This constellation of symptoms is typical of a viral infection. Group A Streptococcus is usually not accompanied by coryza. H. parainfluenzae is not a common cause of pharyngitis. Mycoplasma usually is associated with lower respiratory tract infections. Question: A 9 year-old has been diagnosed with chickenpox. A drug that should be avoided in him is: penicillin.aspirin. Correctibuprofen.sulfa. Explanation: Aspirin is always avoided in the case of viral infections in children and adolescents. The incidence of Reye syndrome is increased if aspirin is given. This is especially true with varicella and influenza infections. The typical constellation of symptoms occurs during a bout of chickenpox and includes nausea, vomiting, headache, excitability, delirium, and combativeness with progression to coma. Since aspirin use has declined sharply, Reye syndrome has too. Question: Which of the following antibiotics may increase the likelihood of photosensitivity? AmoxicillinCephalosporins IncorrectFluoroquinolones CorrectMacrolides Explanation: Many medications can produce a phototoxic reaction when a patient is exposed to sunlight. Antibiotics are especially notable for this. Common antibiotics associated with photosensitivity are tetracyclines, sulfa drugs, and fluoroquinolones. Other common medications that increase photosensitivity are hydrochlorothiazide, diltiazem, selective serotonin reuptake inhibitors, antihistamines, ibuprofen, and naproxen. Question: A patient with diabetes has a right lower leg that has recently become edematous, erythematous, and tender to touch over the anterior shin. There is no evidence of pus, but the leg is warm to touch. What is the most likely diagnosis to consider? Deep vein thrombosis (DVT)Buerger’s diseaseCellulitis CorrectVenous disease Explanation: This description is one of cellulitis. Cellulitis involves an infection of the subcutaneous layers of the skin. It must be treated with an oral antibiotic. This is particularly important to identify early, and aggressively treat in a diabetic because elevated blood sugar levels will make eradication more difficult. Buerger’s disease involves inflammation of the medium sized arteries and does not present on the anterior shin only. DVT seldom presents on the anterior shin. Venous disease does not present acutely as in this situation. Question: A 70 year-old is diagnosed with multiple cherry angiomas. The nurse practitioner knows that: this is a mature capillary proliferation more common in young adults.an angioma occurs as a single lesion.these may bleed profusely if ruptured. Correctthese are precursors of skin malignancies. Explanation: Cherry angiomas are mature capillary proliferations that are more common in middle and older adults. They blanch with pressure and are usually 0.1-0.4 cm in diameter. They are commonly found on the trunk as multiple lesions. Because they are a proliferation of capillaries, they will bleed significantly if they rupture. The bleed is not life threatening, but, in older adults who take aspirin, the bleeding will be worse. Pressure should be held over the ruptured area until bleeding stops. These are not precursors of skin malignancies. APEA TEST BANK-with 100% verified answers-2024-2025 South University Tetanus toxoid only IncorrectTetanus and diphtheria onlyHis primary series will protect him.Tetanus, diphtheria, and acellular pertussis (Tdap) Correct Explanation: More than 10 years has elapsed since this patient’s last tetanus shot. He needs another one. Tdap is specifically indicated for adolescents, older adults, healthcare providers, and pregnant patients who have completed a primary series. Tetanus toxoid is indicated in the rare adult or child who is allergic to the aluminum adjuvant in the Td immunization. Question: Hand-foot-and-mouth disease and herpangina: both produce pustules on oral mucous membranes.potentially can cause orchitis in infected males.are viral infections caused by Coxsackie viruses. Correctwill only occur in the spring and summer. Explanation: These two common viral exanthems are caused by Coxsackie A viruses. Sometimes herpangina is caused by Coxsackie A or B. Ulcerations are produced on the mucous membranes, not pustules. Orchitis can occur in males infected with mumps. Herpangina typically occurs during summer and fall; hand-foot-and- mouth disease (HFM) occurs in late summer and fall. Both produce oral symptoms, but only HFM produces blisters on the palms of the hands and/or soles of the feet. Both are self-limiting viral illnesses. Question: A 6 year-old has been diagnosed with Lyme disease. Which drug should be used to treat him? Doxycycline IncorrectAmoxicillin CorrectAzithromycinCephalexinExplanation: Amoxicillin, doxycycline, and cefuroxime have all been shown to have equivalent efficacy for treatment of Lyme disease. However, because this patient is only 6 years old, he should not be given doxycycline as a first line treatment unless the other regimens are contraindicated. Macrolides, like azithromycin should not be used first line because they are poorly effective at eradicating infection. They may be used in patients who are intolerant of penicillins and cephalosporins and who cannot take doxycycline. First generation cephalosporins like cephalexin are not effective. Question: The nurse practitioner examines a patient who has had poison ivy for 3 days. She asks if she can spread it to her family members. The nurse practitioner replies: “yes, but only before crusting has occurred."“yes, the fluid in the blister can transmit it.” Incorrect“no, transmission does not occur from the blister’s contents.” Correct“no, you are no longer contagious.” Explanation: The skin reaction seen after exposure to poison ivy (or any other skin irritant), takes place because of contact with the offending substance. In the case of poison ivy, the harmful exposure occurs from contact with oil from the plant. The eruptions seen are NOT able to transmit the reaction to other people unless oil from the plant remains on the skin and someone touches the oil. The fluid found in the blisters is NOT able to transmit poison ivy to anyone, only the oil from the plant can do that. After oil has touched the skin, some time must pass for the reaction to occur. Therefore, reaction times vary depending on skin thickness and quantity of oil contacting the skin. Question: Screening for abdominal aortic aneurysm should take place: once for all males aged 65-75 who have ever smoked. Correctonce for all men and women who have hypertension. Incorrectannually after age 75 years for males and females.only if the patient has smoked and has hypertension. APEA TEST BANK-with 100% verified answers-2024-2025 South University Explanation: The prevalence of abdominal aortic aneurysm (AAA) is greater in men than women. American Heart Association and USPSTF recommend screening males once between ages 65-75 years if they have ever smoked. Smoking increases the risk of AAA. The USPSTF does not recommend routinely screening for AAA in women and do not recommend screening for AAA in men who have never smoked. Screening may be considered in men aged 65-75 years if they have a first-degree relative who required repair of AAA. Question: What would be age appropriate anticipatory guidance for the parent of a 9 month-old infant? Keep syrup of ipecac in case of accidental poisoningStranger anxiety will develop by a year of age CorrectYour baby should be able to say 10 words before a yearDiscuss weaning from a bottle Explanation: Weaning from the baby’s bottle takes place about a year of age. This should be discussed at the visit prior to the time this would take place. 9 months is an appropriate time to discuss this. Syrup of ipecac is no longer recommended. Parents and caregivers should be instructed NOT to use this. Stranger anxiety develops about 9 months of age. The average 12 month old says about 2-5 words. Question: How often should hearing be screened with audiometry in older adults? Annually CorrectTwice annuallyAt each visitOnly if symptoms exist Explanation: ACOVE authors recommend annual hearing screen for older adults. The screen should include a questionnaire and handheld audiometry. Screening annually resulted in greater adherence with using hearing assistive devices when hearing problems were identified. Question: Which recommendation below reflects CDC’s recommendation for administration of the zoster vaccine? It should be given to immunocompetent adults age 60 and older. CorrectIt should be given to immunocompetent adults age 65 and older.It can be given regardless of immune status at age 50.It can be given regardless of immune status at age 60. Explanation: The zoster vaccine, to prevent shingles and postherpetic neuralgia from shingles is currently FDA approved for use in immunocompetent adults age 50 or older. CDC’s recommendation is for immunocompetent adults aged 60 years and older. The difference in age recommendation is due to the anticipated shortage of vaccine if it were given to ages 50 years and older. It should never be used to treat shingles. Question: A one year-old patient’s mother reports allergy to gelatin. The mother describes the reaction as “lips swelling and breathing difficulties which necessitated a trip to the emergency department”. Which immunizations should be avoided? Varicella, DTaP, and MMRVaricella and MMR CorrectIPV, DTaP, and Hepatitis BHepatitis A and B, meningococcal Explanation: Varicella and MMR should be avoided until allergy testing can take place. However, according to CDC, this patient should be skin tested prior to administering either of these vaccines to establish sensitivity to gelatin. IF the skin tests are negative, the vaccine can be given as with non-allergic children. Other than MMR and varicella, there are no contraindications to any other US immunizations. APEA TEST BANK-with 100% verified answers-2024-2025 South University Question: APEA TEST BANK-with 100% verified answers-2024-2025 South University Question: A seven year-old entered the clinic one month ago. There was no evidence that he had any immunizations. He was given hepatitis B, DTaP, IPV, varicella, and MMR. If he returns today, which immunizations can he receive? Hepatitis B, Td, Hib, polio, MMRHepatitis B, Td, IPV, MMR, varicella IncorrectIPV onlyHepatitis B, DTaP, IPV, MMR Correct Explanation: The minimum length of time between Hepatitis B, DTaP, IPV, and MMR is one month. Therefore, he can receive all of these today. He should not receive another varicella today. The minimum length of time between immunizations is 3 months if he is less than 13 years of age. Question: A patient who is 62 years old asks if she can get the shingles vaccine. She has never had shingles but states that she wants to make sure she doesn’t get it. What should the nurse practitioner advise? The immunization will protect you from acquiring shingles.You are not old enough to receive the immunization.The immunization is offered only to those who have had shingles.You are eligible to receive it but you still may get shingles. Correct Explanation: Patients must be at least 50 years old to receive the shingles immunization. It is generally well tolerated but provides protection from shingles in 50-64% of patients. The incidence of post-herpetic neuralgia is decreased up to 65% after immunization. The patient still may develop shingles after receiving the immunization. The vaccine may be offered regardless of whether the patient has history of shingles. However, since it is a live vaccine, it may be contraindicated because of steroid use or immune status. Question: A 58 year-old patient has an annual exam. A fecal occult blood test was used to screen for colon cancer. Three were ordered on separate days. The first test was positive; the last two were negative. How should the nurse practitioner proceed? Re-screen in one year.Perform a fourth exam.Refer him for a colonoscopy. CorrectExamine him for hemorrhoids. Incorrect Explanation: A fecal occult blood test is performed multiple times on different days because tumors don’t consistently excrete blood. The reason multiple are performed is to increase the likelihood of identifying blood. The patient needs to have a colonoscopy performed for examination of the colon. The standard of practice is to refer all positive colon cancer screens for colonoscopy. Question: A pregnant patient is concerned because her 12 month-old needs an MMR immunization. What should the NP advise this patient? The MMR is a dead virus; therefore, there is no risk to the mother.Have the child wait until Mom delivers.Have the child wait until Mom is in her 28th week or later before immunization.MMR immunization presents no risk to the child’s mother. Immunize now. Correct Explanation: MMR immunization in the child presents no risk to the pregnant mother. MMR is actually attenuated, also thought of as an inactivated form of the virus. Therefore, there is no risk to her. If the mother were pregnant, she should not be vaccinated. If she is contemplating pregnancy, she should avoid MMR for 4 weeks following immunization. Question: APEA TEST BANK-with 100% verified answers-2024-2025 South University A patient is 86 years-old and functions independently. He has hypertension, hyperlipidemia, BPH, and flare-ups of gout. His last colonoscopy was at age 76 years. What should he be advised about having a colonoscopy? Colonoscopy is the preferred method for screening in the elderly IncorrectColonoscopy is ill advised in the elderlyThe patient should consider an alternative screening method CorrectScreening is not necessary after age 80 years Explanation: Colonoscopy enables the examiner to visualize the entire colon and is a superior screening tool for colorectal cancer, but there are significant risks of bleeding and perforation in older adults. Additionally, the colon prep can produce massive shifts in electrolytes that can increase the likelihood of arrhythmias, weakness, and falls. As a general rule, patients with a life expectancy of less than 5 years should forego colorectal cancer screening via colonoscopy. An alternate method such as stool for occult blood could be considered in this patient. Prior to any screening test such as colonoscopy, consideration must be given to the patient’s overall health status, colorectal cancer risk, and desire to pursue treatment if cancer or disease is diagnosed. Question: A criterion for medication choice in an older adult is: long half-life to prevent frequent dosing.dosing of 3-4 times daily.pill color and shape for easy identification.half-life less than 24 hours. Correct Explanation: Many factors go in to prescribing for older adults. Some important safety criteria include established efficacy, low adverse event profile, half-life less than 24 hours with no active metabolites. Active metabolites would produce a longer effect of the drug in the patient. Dosing of a medication 3-4 times daily invites dosing and medication errors. Once or twice daily dosing is ideal. Pill color and shape is never a criterion for prescribing. Patients who are cognitively able will recognize the color, shape, and size of pills they take on a regular basis. Question: An oral antifungal agent is commonly used to treat tinea unguium. The difficulty in treating an older adult with this is infection is: absorption of the medication. Incorrectapplying the medication twice daily.tolerability of the medication. Correctrelative ineffectiveness of oral agents. Explanation: The most efficacious agents used to treat toenail fungus are the oral antifungal agents. They must be taken daily for 8-16 weeks (or longer) for adequate length of treatment. Additionally, the real difficulty lies in the ability of the elderly patient’s liver to handle this medication. Oral anti-fungal agents require great amounts of the liver’s resources for metabolism. A topical agent or toenail removal may be a better choice for an older adult. Question: Head circumference should be measured until a child has attained: 12 months of age. Incorrect18 months of age.24 months of age.36 months of age. Correct Explanation: A child’s head is usually measured at each periodic well child visit until he has attained the age of 3 years old. During the first three years of life, the head grows and its growth should be monitored for adequacy. APEA TEST BANK-with 100% verified answers-2024-2025 South University The tape measure should encircle the head at the largest point above the ears. These measurements are recorded on a growth chart so that changes in head circumference can be followed and percentile for age and rate of growth can be determined. Question: What is true regarding the “shingles” vaccine given to adults at or after age 50? It is a weakened form of the chickenpox virus.It is the same as the chickenpox virus.It contains significantly more virus than the chickenpox vaccine. CorrectIt is not related to the chickenpox immunization at all. Incorrect Explanation: The “shingles” (Herpes zoster) vaccine contains 14 times the number of plaque-forming units of virus than the varicella vaccine. The immunization has reduced the incidence of shingles and post-herpetic neuralgia in adults who received it. The vaccine is recommended by CDC for all immunocompetent adults who are 60 years or older. It is given once. The FDA has approved use of this vaccine at age 50 years. The vaccine is only used to prevent shingles. It is not used to treat shingles or post-herpetic neuralgia. Question: What is the most common nutrition syndrome in elderly patients? Undernutrition CorrectOver nutritionMalnourishmentVitamin B-12 deficiency Incorrect Explanation: Some elderly patients have great challenges associated with eating and maintaining weight. They may be edentulous or have anorexia. Weight loss is associated with greater mortality in older adults than in patients who have not had recent weight loss. Clinically significant weight loss is usually considered to be about 4- 5% of total body weight within 6-12 months. In older patients who do lose weight, they are less likely to gain it back than younger adults. This also increases risk of mortality. Question: A mammogram in a healthy 50 year-old female patient is considered to be an example of: primary prevention.secondary prevention. Correcttertiary prevention.quaternary prevention. Explanation: This is an example of secondary prevention. Secondary prevention is represented by screenings intended to identify early course of a disease. In this example, a mammogram is intended to identify early breast cancer. Question: A 12 month-old is here today to be vaccinated with varicella. A patient’s mother reports that her 12 month- old child was exposed to chickenpox about a week ago. The NP should recommend that he: be given the vaccine. Correctwait 4 weeks to receive the vaccine. Incorrectnot receive varicella today because he is not old enough.receive MMR only today, not varicella. Explanation: The patient who reports recent exposure to an infectious disease should be vaccinated today unless something else contraindicates the immunization. Since he is 12 months old, he is old enough to receive varicella and MMR. Varicella should be given on the same day as MMR or the two vaccines must be separated by at least 4 weeks. Question: A side effect of DTaP that should be reported is: temperature of 103F. Correctvomiting.non-stop crying (3h or more).injection site soreness lasting greater than 2 days. Incorrect APEA TEST BANK-with 100% verified answers-2024-2025 South University No studies have demonstrated specific benefit to daily consumption of multivitamins in older adults. However, a daily multivitamin supplement ensures adequate intake of essential vitamins and so should be considered for older adults. Question: A 7 year-old enters the nurse practitioner clinic. There is no evidence that he has received any immunizations. What should be administered today? Hepatitis B, Tdap, Hib, IPV, MMRTd, Hib, IPV, varicellaHepatitis B, Tdap, IPV, varicella, MMR CorrectHepatitis B, IPV, varicella, MMR Explanation: This child will be placed on CDC's catch-up schedule (a copy can be downloaded from CDC's website). Because of his age, he does not need a Hib immunization. He does need all of the immunizations listed in choice C. Question: An octogenarian asks the nurse practitioner if it is OK for him to have an alcoholic beverage in the evenings. There is no obvious contraindication. How should the nurse practitioner respond? Yes, but not more that 4 days per week.Yes, but not more than 1-2 drinks per day. CorrectNo, you will increase your risk of falling and injury.It depends on the type of alcohol you would like to consume. Explanation: A good rule of thumb for alcohol consumption in older adults is no more than 1-2 drinks/day after age 65 years. If the patient is cognitively impaired, abstinence is recommended. The type of alcohol is not of great importance. Beer, wine, and hard liquor all contain alcohol with the potential to impair older adults. The reason alcohol should be limited or avoided is because of decreased lean body mass and decreased total body water in aging bodies. Question: A nurse practitioner examined a patient who had been bitten by her husband during an assault. There were numerous bite marks and lacerations on the patient’s forearms. The nurse practitioner sutured the lacerations, though this was contraindicated because of the highly infectious nature of human bites. The patient suffered no ill effects after suturing. How can this be described? This is negligence. CorrectThis was a fortunate situation for the patient.This is malpractice.This is poor judgment and malpractice. Explanation: This is not malpractice. This is negligence. Negligence occurs when one fails to exercise the care that a reasonable person would exercise. Injury does not have to occur for negligence to occur. Human bites, known to be dirty bites with high probability of infection should not be sutured. Malpractice is usually described as having multiple elements that all must be satisfied for malpractice to occur. There must be a duty, a breach of the duty, and a subsequent injury due to the breach. Comparison of performance is based on the standard of care delivered by nurse practitioners. Question: A nurse practitioner is working in a minor care area of an emergency department. An illegal immigrant has a puncture wound caused by an unknown sharp object in a trash container. A dirty needle is suspected. The nurse practitioner: should administer a tetanus injection only since the patient has no medical insurance. Incorrectshould prescribe appropriate medications for HIV exposure even though the nurse practitioner knows the patient can’t afford them. Correctshould not mention the possibility of HIV exposure from a dirty needle to the patient.can offer to buy the HIV medications for $50 with the professional discount at the pharmacy next door. Explanation: APEA TEST BANK-with 100% verified answers-2024-2025 South University The standard of care followed by the nurse practitioner should not depend on whether the patient has insurance or not. It is unethical to not properly inform the patient of risks he may have been exposed to from the puncture wound. Offering to buy the medications for the patient is very noble but is not a sustainable practice. The nurse practitioner should prescribe the medications as for anyone with possible HIV exposure and refer to social services or a community referral agency that can help this patient acquire the appropriate medications. Question: Laura admits to Anne that she keeps the medications and takes them occasionally. She states that sometimes she gives them to patients who can’t afford these medications. What is Anne’s first professional responsibility? Report this to the police.Report Laura’s actions to the state board of nursing. CorrectReport Laura’s actions to the state board of pharmacy.Anne has no professional responsibility for Laura’s practice. Explanation: Anne has a responsibility to report unsafe practice to the state board of nursing. This should be done first. She also has a responsibility to report a potentially impaired healthcare provider, especially if she observes this. Since there is no specific evidence to indicate what Laura has done with the medications she took from patients, theft will be hard to establish. State Board of Pharmacy will probably become involved in the investigation but their role varies from state to state. Question: A nurse practitioner gives a patient 2 weeks of sample medications that will be taken once daily by the patient. The sample medications are packaged by the drug manufacturer. The nurse practitioner’s actions are an example of: prescriptive rights.prescriptive authority.dispensing. Correctadministering. Explanation: Although definitions vary from state to state, a reasonable definition of dispensing is the legal right to select (as from stock) and/or label a medication to be self-administered by a patient. Most states allow registered nurses to dispense medications provided they are pre-packaged. A registered nurse is not allowed to prescribe the medication being dispensed. Administering usually refers to the act of giving a single dose of a medication to a patient. Medication administration may be performed by nurses in all states. Question: A patient that you are caring for in your clinic has Medicare Part B. What does this mean? The federal government will pay for his visit to your clinic today.His Medicare benefit covers outpatient services. CorrectHe will have a co-pay for his visit today.His prescriptions will be partly paid for today. Explanation: Part B pays the examiner (NP, PA, MD, etc.). Part B of Medicare pays for outpatient care, ambulatory surgery services, x-rays, durable medical equipment, laboratory, and home health. Part B is an option that Medicare recipients can pay for with a monthly option. This charge is based on income. Since there is an initial co-pay, the federal government’s insurance plan may NOT pay for his visit to your clinic today. Question: An elderly male with moderately severe dementia presents with his caregiver daughter. His BMI is 18. His clothes have food stains on them and he looks as though he hasn’t been bathed in days. How should the nurse practitioner handle this? The NP should comment to the daughter about his poor care.The NP should report this as potential elder abuse. CorrectThe patient should be asked about his care.The daughter should be asked about the type of care he receives. Incorrect Explanation: APEA TEST BANK-with 100% verified answers-2024-2025 South University This patient presents as though he is being poorly cared for and mistreated. This occurs in about 3-8% of the adult population in the United States. There is no evidence that the patient has been physically abused, but he obviously suffers from neglect. This is a form of elder abuse just as physical, sexual, psychological, or financial abuse is. Elderly patients with dementia are those who suffer abuse most frequently. Question: Prescriptive authority: is a right of every practicing nurse practitioner.varies from state to state. Correctis not part of the consensus model.is dependent on certification. Explanation: Prescriptive authority refers to the ability of a nurse practitioner to work within the legal scope of practice in all 50 states. Although nurse practitioners are allowed to prescribe in all 50 states, there is great variability in how this occurs. Some states allow broader prescriptive authority, other much narrower authority. Question: A nurse practitioner examined a patient who had been injured by a cat. There was a 4 centimeter gaping laceration on the patient’s forearm. The nurse practitioner sutured the laceration. The patient subsequently became infected, needed hospital admission, and required IV antibiotics with incision and drainage. How can this situation be characterized? This is a clinical judgment with an unexpected complication.The nurse practitioner’s actions followed standard of care.The act of suturing this type of wound represents malpractice. CorrectThis is poor judgment, but not malpractice. Explanation: This is malpractice. Cat bites, known to be dirty bites with high probability of infection, should not be sutured. Malpractice is usually described as having multiple elements which all must be satisfied for malpractice to occur. There must be a duty, a breach of the duty, and subsequent injury due to the breach. Comparison of performance is based on the standard of care delivered by nurse practitioners. Question: The next two questions relate to each other. 18. Anne and Laura work as nurse practitioner (NP) partners in an NP practice. Anne learns that Laura frequently changes patient’s narcotic prescriptions to a different dosage, and then requests that the patient give her the remaining narcotic medications. Some patients have called Anne aside to tell her of this. How can Laura’s actions best be characterized? Laura is committing theft.Laura’s behavior is not illegal but is unethical.Requesting the patient’s medication is within Laura’s scope of practice.This appears to be diversion of medications by Laura. Correct Explanation: Diversion of medications occurs when medications intended for one person, end up in the hands of another person. This is particularly serious when the medication involved is a narcotic. The question stem does not state what Laura is doing with the medications, but because of the nature of the action and the fact that it involves narcotics, Laura should not be engaging in this behavior because of the look of impropriety. Question: You are volunteering at a clinic that cares for homeless patients. What’s the most important aspect of a patient’s first visit? A complete head to toe examEstablish trust CorrectTake an excellent history IncorrectAsk about problems with alcohol Explanation: APEA TEST BANK-with 100% verified answers-2024-2025 South University her employer, patient, or state board of nursing as long as the performance of her job does not impose unnecessary risk to anyone. Question: A nurse practitioner’s scope of practice is influenced by a number of factors. Which one does not influence scope of practice? Code of ethicsState and federal laws governing practiceCourt of law CorrectEducational preparation Incorrect Explanation: The scope of practice for nurse practitioners is established legally, ethically, and by boards of nursing and professional organizations. Scope of practice sets the boundaries and indicates what is permitted legally, etc. Scope of practice is not influenced by court of law. Scope of practice is determined by state statutes, state nursing boards, common practice in a locale, educational preparation, and others. Scope of practice can vary from state to state. Question: A nurse practitioner is working in a minor care clinic. She realizes that a patient with a minor laceration does not have insurance and is using his brother’s insurance information today so that his visit will be covered. How should she proceed? She should let him know that she knows what he is doing.She should ignore this and proceed to suture his wound.She should let the clinic's business office know what is happening. CorrectShe should not suture his wound and ask him to leave. Explanation: The nurse practitioner cannot ignore the fact that this patient is attempting to defraud the clinic and insurance company in order to receive free care. If she does not let the business office know, she is a party to the fraud. She should let the business office know what is happening and have the patient present documents verifying that he is who he states that he is. If he cannot, he can still receive care if he is willing to pay for it. An alternative site to receive care should be offered to him. Question: What would be the study of choice to determine the cause of a cluster of adult leukemia cases found in an isolated area of a rural state? Randomized clinical trialCohort studyCase series IncorrectCase control Correct Explanation: A case control study would be ideal for discovering the cause of this situation. Case control looks at “what happened”? It would identify those subjects who have leukemia and would identify a control group of adults from the area who did not have leukemia. Both groups would be analyzed for characteristics or risk factors that were present in the “case” group but not the “control” group. This is an observational study. Question: Standards of practice are established to: regulate and control nurse practitioner practice. Correctlimit liability of nurse practitioners.protect nurse practitioners from frivolous law suits.promote autonomous practice. Explanation: Standards of practice for all professionals (nurses, physicians, dentists, etc.) are established to regulate and APEA TEST BANK-with 100% verified answers-2024-2025 South University control practice. They are intended to provide accountability for professionals and to help protect the public from unethical behavior and unsafe practice. Question: The nurse practitioner decides to study a group of patients who are trying to quit smoking. They all will be taking the same type of medication for 42 days to help them stop smoking. The patients have agreed to return to the clinic once weekly for the study’s duration. This type of study design is termed: experimental study.cohort study. Correctcase control study.controlled trial. Explanation: A cohort study describes an observational study that is prospective in nature, such as the case with this group of smokers. Cohort studies usually ask the question, “what will happen?” A case control study looks backward in time (retrospective). Case control studies usually ask the question, “what happened?” A controlled trial is an experimental study, not observational. Question: The nurse practitioner is examining an elderly patient with dementia. She is noted to have bruises on her arms and on her posterior thoracic area. The nurse practitioner suspects elder abuse, but cannot be certain. The daughter of this elderly patient is her caregiver. The daughter is a patient of the nurse practitioner. What should the nurse practitioner do? Do not report the abuse until the NP is certain of it.Rule out elder abuse since her daughter is the caregiver.Report it to the appropriate authorities. CorrectAsk the daughter if she is abusing her mother. Explanation: Actual or suspected elder abuse is reportable in all 50 states and healthcare providers are all mandatory reporters. Most elders are abused by their caregivers; particularly when the elder is demented. If the nurse practitioner asks the patient’s daughter if she is abusing her mother it will likely result in a negative response by the daughter. Additionally, the daughter is not likely to seek this nurse practitioner’s help in the future because of fear of retribution. Question: The Medicaid funded health program is: funded with premiums from participants.unlimited on the number of adult visits.funded by both state and federal governments. Correctbasically the same from state to state. Explanation: Medicaid is state run and specific to each state. The state programs are funded by a combination of state and federal funds. Most states have limits on the number of adult visits. Some states have no limits on visits for children. Participants generally do not pay premiums like Medicare recipients pay. Question: A nurse practitioner has worked for a large hospital as an RN. As a new nurse practitioner, she has developed a nurse practitioner managed clinic for hospital employees and is employed by the hospital. This nurse practitioner is described as a(n): intrapreneur. Correctentrepreneur.risk taker.nurse specialist. Explanation: An intrapreneur is someone who is able to carve out a specialty role within an existing organization, healthcare setting, or business/industrial setting. An entrepreneur is someone who assumes the financial and personal risks of owning and operating a business. APEA TEST BANK-with 100% verified answers-2024-2025 South University Question: Which study listed below is considered an experimental study? Case series IncorrectCross-sectional studyCohort studyMeta-analysis Correct Explanation: Observational studies are studies where subjects are observed. No intervention takes place with them. Examples of these are found in the first three choices. A meta-analysis takes published information from other studies and combines the information to arrive at a conclusion. Although a meta-analysis can use observational studies, these should be reported separate Question: Certification: is required by all 50 states.validates competence. Correctis recognized by all 50 states. Incorrectprovides for title protection. Explanation: Certification is a process used to validate competence of an individual in an area of specialty. For example, certification exams are available for nurse practitioner graduates in the areas of family, pediatrics, women’s health, adult-gerontological. While Medicare requires certification in order for a nurse practitioner to independently bill, it is not required for reimbursement by all organizations. Certification is not required by all states to practice as a nurse practitioner. Title protection is provided by licensure, not certification. Question: Who certifies nurse practitioners? The state where they practiceState board of medicine or nursingA state board of nursingA nurse practitioner certifying body Correct Explanation: The 2 certifying bodies for adult, adult-gero and family nurse practitioners in the United States are American Nurses Credentialing Center (ANCC) and American Association of Nurse Practitioners (AANP). State boards of nursing grant a license to nurse practitioners in the state where they practice. Most states require certification in order to become licensed. Question: The name given to subjects in a research study who do not have the disease or condition being studied, but who are included in the study for comparison are: studied subjects.controls. Correctcase series.cross sectionals. Incorrect Explanation: Studied subjects are those members of a study who have a specific disease or condition of interest or who are receiving a specific treatment. Case series may refer to an observational study where a group of patients with interesting characteristics are studied. Cross sectional is a type of observational study where a particular characteristic is studied at one time rather than over time. Controls are commonly employed in many types of research studies. Question: A nurse practitioner has agreed to participate in the Medicare health insurance program. Medicare paid 80% of the charges billed for a clinic visit. What can be done about the other 20% that is owed? APEA TEST BANK-with 100% verified answers-2024-2025 South University states that she met her sister in the clinic today. The neighbor replies, “Don’t you work in an HIV clinic?” How can this situation be characterized? This is negligence.This is a breach of confidentiality. CorrectThis is not a breach of confidentiality.The NP has no liability. Explanation: This is a breach of the patient’s confidentiality. The neighbor’s sister is the NP’s patient. The NP is bound to confidentiality regarding protected health information for any of her patients. Even though the NP did not acknowledge that her sister was a patient, she breached patient confidentiality by telling the patient’s sister that she had met her in the clinic. This is different than telling her sister she met her at the grocery store. Question: What is the usual age for vision screening in young children? 2 years3 years Correct4 years5 years Explanation: Initial vision screening should take place at 3 years of age. If the child is not cooperative, screening should be attempted 6 months later. If the child is still not cooperative at 3.5 years, it should be attempted at 4 years. Generally, children are cooperative at 4 years of age. The usual vision of a 3 year-old is 20/50. Question: A patient has nasal septal erosion with minor, continuous bleeding. There is macerated tissue. What is a likely etiology? Improper use of a nasal steroidChronic sinusitisSevere allergic rhinitisCocaine abuse Correct Explanation: The nasal septum separates the right from left nostrils. It is made of thick cartilage and is covered with mucous membrane. It can be injured from foreign substances that contact it, like cocaine. A nasal septal erosion or perforation should always be assumed to have been from sniffing toxic substances in the nose, not nasal steroids. Question: The nurse practitioner performs a fundoscopic exam on a patient who has recently been diagnosed with hypertension. What is the significance of AV nicking? This is an incidental finding.This is indicative of long standing hypertension. CorrectThe patient should be screened for diabetes. IncorrectThe patient should be referred to ophthalmology. Explanation: Normally, veins are larger than arteries in the eyes. The vessels in the eyes are particularly susceptible to increased blood pressure. AV (arterio-venous) nicking can be observed as arteries cross veins when the arteries have narrowed secondary to hypertension. Generally, AV nicking takes time to develop and would be expected in patients with long standing hypertension; especially when it is poorly controlled. Cotton wool exudates should prompt the examiner to screen for diabetes. An ophthalmology referral is not required at this point for AV nicking. In severe hypertension, the retina can become detached. Question: Which statement about serous otitis media is correct? This usually needs treatment with antibiotics. IncorrectThis can be diagnosed with pneumatic otoscopy. CorrectSerous otitis media can produce a sensorineural hearing loss.Otitis media and serous otitis are frequently associated with fever. Explanation: APEA TEST BANK-with 100% verified answers-2024-2025 South University Serous otitis media (SOM) is also called otitis media with effusion (OME) or "glue ear". OME occurs when there is fluid (non-infectious) in the middle ear. This prevents normal mobility of the tympanic membrane and creates a conductive hearing loss. Pneumatic otoscopy is the primary non-invasive diagnostic method because it has a high sensitivity and specificity. It may be present before otitis media develops, or it may follow resolution of otitis media. OME is far more common than otitis media and is not associated with systemic symptoms like fever. Acute otitis media (AOM) describes infected fluid in the middle ear. Question: A medication considered first line for a patient with allergic rhinitis is a: decongestant.non-sedating antihistamine.leukotriene blocker.topical nasal steroid. Correct Explanation: Allergy and asthma guidelines in the US recommend topical nasal steroids first line for management of symptoms of allergic rhinitis. A non-sedating antihistamine can be added to manage unresolved symptoms after a nasal steroid has been initiated. Antihistamines work well when the predominant symptoms are thin, clear nasal discharge. These can be safely used in combination for management. Decongestants are not recommended as lone agents because they have no effect on the underlying allergic mechanisms. They work well in combination with antihistamines and nasal steroids for congestion. Sedating antihistamines are usually avoided for allergic rhinitis because of safety concerns. Question: What clinical finding necessitates an urgent referral of the patient to an emergency department? A fiery red epiglottis CorrectSudden onset of hoarsenessPurulent drainage from the external canalTragal tenderness Explanation: A finding of a fiery red epiglottis signals epiglottitis. Since airway obstruction can be rapid with epiglottitis, immediate referral to an emergency department is warranted. Sudden onset of hoarseness does not signal a specific emergency situation. Purulent drainage from the external canal may signify a ruptured tympanic membrane or otitis externa. Tragal pain is significant of otitis externa. Question: A patient stated that his ears felt stopped up. He pinched his nose and blew through it forcefully. The nurse practitioner diagnosed a ruptured left tympanic membrane. What would indicate this? Bright red blood in the left external canal CorrectPain in the left and right earsClear fluid in the left external canalAbsence of hearing in the left ear Explanation: This patient ruptured his tympanic membrane (TM) traumatically from excessive pressure when he pinched his nose and blew out through it at the same time. It is common to find bright red blood (not clear fluid), but not active bleeding, in the external canal of the affected ear. He may experience pain in the affected ear, but this alone would not be indicative of a ruptured TM. Hearing may be diminished in the affected ear but should not be absent. Patients usually describe hearing as muffled. Question: A 70 year-old patient in good health is found to have a large, white plaque on the oral mucosa of the inner cheek. There is no pain associated with this. What is a likely diagnosis? CheilitisAphthous ulcerSjögren's syndromeLeukoplakia Correct Explanation: The etiology of this white plaque is unclear from the given information, but it cannot be cheilitis. This affects the lips. It cannot be an aphthous ulcer because this is painful. Sjögren's syndrome does involve the mucous membranes but manifests itself as dry mouth, not a plaque or lesion. The differential diagnosis for a white oral plaque should include oral leukoplakia, a premalignant lesion. This is often related to HPV, human papilloma virus. Risk factors include smokeless tobacco. Others in the differential include oral hairy leukoplakia (seen almost exclusively in patients with HIV), squamous cell carcinoma, and malignant melanoma. It may also be a completely benign growth, but this can only be established after biopsy. APEA TEST BANK-with 100% verified answers-2024-2025 South University Question: Which of the following will decrease the risk of acute otitis media in a 6 month old? Cigarette smoke exposureBreastfeeding CorrectSucking on pacifiersVitamin D supplementation Question: Swimmer’s ear is diagnosed in a patient with tragal tenderness. What other symptom might he have? Otitis mediaHearing loss IncorrectOtic itching CorrectFever Explanation: Swimmer’s ear is termed otitis externa. It represents an infection of the external canal. This is characterized by tragal tenderness with light touch of the tragus on the affected side. Fever does not occur because this is a superficial infection. It is treated with a topical agents: an antibiotic and steroid placed in the external canal. Question: In a patient with mononucleosis, which laboratory abnormality is most common? Lymphocytosis and atypical lymphocytes Correct Question: A common complaint in older patients who have cataracts is: sensitivity to sunlight. Correctpoor peripheral vision.increased incidence of falls. Incorrecteye pain in the affected eye. Explanation: Loss of peripheral vision and eye pain is typical in patients with glaucoma, not cataracts. Patients with cataracts may be at higher risk for falls because they have difficulty seeing, however, this is not a common complaint of patients who have cataracts. Question: A 70 year-old female states that she sees objects better by looking at them with her peripheral vision. She is examined and found to have a loss of central vision, normal peripheral vision, and a normal lens. This best characterizes: glaucoma.cataracts.macular degeneration. Correctdetached retina. Explanation: Macular degeneration presents most commonly with a loss of central vision. The macula is the central part of the retina. As it degenerates, central vision is lost. Questions should be asked about the rate of loss of vision. Reports of rapid vision loss require urgent ophthalmologic evaluation. Known risk factors are age greater than 50 years with the greatest prevalence over age 65, smoking, family history, and history of stroke, MI, or angina. Question: Arcus senilis is described as: normal in people over 50 years of age. Correctcopper deposits in the cornea.loss of central vision.degeneration of the arcus and obstruction of tear ducts. Incorrect Explanation: Arcus senilis describes an arc or circle around the cornea that is common in older adults. The circle is due to deposition of lipids in the cornea but is not necessarily due to hypercholesterolemia. However, when this is seen in young adults, it is termed arcus juvenilis, and is often associated with lipid abnormalities. Question: A patient who is 52 years old presents to your clinic for an exam. You notice a yellowish plaque on her upper eyelid. It is painless. What should the NP assess? APEA TEST BANK-with 100% verified answers-2024-2025 South University needs an antibiotic.probably has a viral infection.probably has just had acute otitis media. Correcthas cloudy fluid in the middle ear. Incorrect Explanation: Otitis media with effusion (OME) frequently precedes or follows an episode of acute otitis media. This condition should not be treated with an antibiotic since the middle ear fluid is not infected. However, the fluid acts as a medium for bacterial growth. Question: A patient with environmental allergies presents to your clinic. She takes an oral antihistamine every 24 hours. What is the most effective single maintenance medication for allergic rhinitis? AntihistamineDecongestantIntranasal glucocorticoids CorrectLeukotriene blockers Incorrect Explanation: These agents are particularly effective in the treatment of nasal congestion and would be a good choice for the patient in this scenario. Intranasal glucocorticoids are effective in relieving nasal congestion, discharge, itching, and sneezing. A trial of stopping the oral antihistamine could be tried in this patient. Symptoms would determine whether the antihistamine should be resumed. Question: A patient describes a sensation that "there is a lump in his throat". He denies throat pain. On exam of the throat and neck, there are no abnormalities identified. What is the most likely reason this occurs? Factitious sore throatLymphadenitisGlobus CorrectEsophageal motility disorder Incorrect Explanation: The term used to describe the sensation of "a lump in my throat", or the feeling that there is a foreign body in the throat, is globus. It is not associated with sore throat or pain. The most common cause of globus is GERD or other disorders of the upper esophageal tract. If there is no actual foreign body or abnormality, then other etiologies may be psychologic or psychiatric disorders. Question: The most common complication of influenza is: cough.bacterial pneumonia. Correctviral pneumonia. Incorrectbronchitis. Explanation: Pneumonia is the most common complication of influenza; bacterial pneumonia is the most common form. Streptococcus pneumoniae is the most common bacterial pathogen. 25% of deaths associated with influenza are related to pneumonia. Clinical presentation of pneumonia as a complication of influenza is characterized by worsening of symptoms after an initial period of improvement for 1-3 days. Fever, cough, purulent sputum predominate. Cough is a symptom of influenza, not a complication. Bronchitis might be part of the differential of influenza, however, fever is uncommon in bronchitis. Question: Which of the following is most likely observed in a patient with allergic rhinitis? Exacerbation of symptoms after exposure to an allergen CorrectNasal congestion and sneezing IncorrectPost nasal drip and sore throatWorsening of symptoms during a sinus infection Explanation: Allergic rhinitis is usually diagnosed on clinical presentation and history. The diagnosis is appropriately made when allergic rhinitis symptoms are reproducible after exposure to the suspected allergen, like pollen. APEA TEST BANK-with 100% verified answers-2024-2025 South University Nasal congestion, sneezing, post-nasal drip and sore throat are not exclusive to allergic rhinitis. Finally, sinusitis does exacerbate allergic rhinitis symptoms. Question: A nurse practitioner performs a fundoscopic exam. He identifies small areas of dull, yellowish-white coloration in the retina. What might these be? Cotton wool spots CorrectMicroaneurysmsHemorrhagesExudates Explanation: These are cotton wool spots. They are due to swelling of the surface layer of the retina. Swelling occurs because of impaired blood flow to the retina. The most common causes of cotton wool spots are diabetes and high blood pressure. A microaneurysm is the earliest manifestation of a diabetic retinopathy. These appear as small round dark red dots on the retinal surface. Exudates are an accumulation of lipid and protein. These are typically bright, reflective white or cream colored lesions seen on the retina. Question: The throat swab done to identify Streptococcal infection was negative in a 12 year-old female with tonsillar exudate, fever, and sore throat. What statement is true regarding this? A second swab should be done to repeat the test.The patient does not have Strept throat.The patient probably has mononucleosis.A second swab should be collected and sent to microbiology. Correct Explanation: A second swab is collected, but it is not used to repeat the test. The second swab is sent to microbiology for culture. The sensitivity varies in office Strept tests. Some are as low as 50% and a second swab should be collected. If beta-hemolytic Strept organisms are grown out, then the patient can be diagnosed with Streptococcal infection. Question: A 6 month-old infant has a disconjugate gaze. The nurse practitioner observes that the 6 month old tilts his head when looking at objects in the room. Which statement is true? Nystagmus will be present.The infant will have an abnormal cover/uncover test. CorrectThe patient’s vision is 20/200. IncorrectHe needs a CT to rule out an ocular tumor. Explanation: The cover/uncover test is used to assess strabismus, a common cause of disconjugate gaze. Strabismus represents a nonparallelism of the visual axis of the eyes. This results in the inability of both eyes to focus on the same object at the same time. At 6 months of age, a disconjugate gaze and tilting of the child’s head is a red flag. This child needs referral to ophthalmology. While an ocular tumor could be present, this is unlikely and not the action that should be taken today Question: Group A Strept pharyngitis: is characterized by a single symptom.can be accompanied by abdominal pain. Correctusually does not have exudative symptoms.is commonly accompanied by an inflamed uvula. Incorrect Explanation: Group A Streptococcus is usually characterized by multiple symptoms with an abrupt onset. Sore throat is usually accompanied by fever, headache. GI symptoms are common too; nausea, vomiting and abdominal pain are usual. Even without treatment, symptoms usually resolve in 3-5 days. APEA TEST BANK-with 100% verified answers-2024-2025 South University Question: A 3 year-old has fluid in the middle ear that does not appear infected. The eardrum appears normal. This is referred to as: serous otitis media.acute otitis media.otitis media with effusion. Incorrectmiddle ear effusion. Correct Explanation: Middle ear effusion refers to the presence of fluid in the middle ear. This is present in both otitis media with effusion and acute otitis media. Since the eardrum appears normal and the fluid does not appear infected, there is no reason to suspect otitis media, acute or with effusion. Another name for otitis media with effusion is serous otitis media. Other terms for this are secretory or nonsuppurative otitis media. Question: Conjunctivitis: produces blurred vision in the affected eye. Incorrectusually begins as a viral infection. Correctproduces anterior cervical lymphadenopathy.is common in patients who are nearsighted. Explanation: Conjunctivitis or “pink eye” usually begins as a viral infection. As the conjunctiva becomes irritated, the eye is rubbed and fingers introduce bacteria. A secondary bacterial infection develops. Conjunctivitis produces a red (or pink) eye, but should never produce blurred vision. A patient with a red eye and blurred vision should be referred to ophthalmology. The pre-auricular nodes may be palpable when a patient has conjunctivitis, not the anterior cervical ones. Question: A patient who presents with a complaint of sudden decreased visual acuity has a pupil that is about 4 mm, fixed. The affected eye is red. What might be the etiology? StrokeBrain tumorGlaucoma CorrectCataract Explanation: This patient needs urgent referral to ophthalmology. While this is a relatively unusual patient in primary care, the primary care clinician must be able to recognize this patient and the need for urgent referral. In a patient with acute angle closure glaucoma, the patient is usually ill appearing, may have nausea and vomiting. This scenario should prompt urgent referral. Question: Which long-acting antihistamine listed below is sedating? LoratadineCetirizine CorrectAzelastine IncorrectFexofenadine Explanation: Cetirizine (Zyrtec) is NOT non-sedating. In older patients, the usual dose is decreased from 10 mg daily to 5 mg daily. This is due to the sedative effect of cetirizine. It should be dosed at nighttime to minimize the initial sedative action. It has a prolonged geriatric half-life. Caution is advised. Fexofenadine is the least sedating of the oral agents listed. Question: A 2 year-old has a sudden onset of high fever while at daycare. The daycare attendant describes a seizure in the child. The child is brought to the clinic; neurologically he appears normal. His body temperature is 99.9 degrees F after receiving ibuprofen. He is diagnosed with otitis media. How should the nurse practitioner manage this? APEA TEST BANK-with 100% verified answers-2024-2025 South University Newborns may present with pneumonia and/or conjunctivitis. The most common clinical feature is conjunctivitis that occurs 5 to 14 days after delivery. It is characterized by swelling of the lids and a watery discharge that becomes mucopurulent. The conjunctivae are erythematous. This must be treated orally because topical treatment is not effective. The drug of choice in infants is oral erythromycin 50 mg/kg/day in divided doses for 14 days whether treating pneumonia or conjunctivitis. Question: A 4 month-old infant has thrush. The mother is breastfeeding. She reports that her nipples have become red, irritated, and sensitive. What should the nurse practitioner advise the mother of this baby to treat thrush? Have the mother exercise good hygiene of her nipplesAdminister an oral anti-fungal suspension to the motherAdminister an oral anti-fungal suspension to the infant IncorrectTreat the infant with an oral anti- fungal suspension and the mother’s nipples with a topical anti-fungal agent Correct Explanation: If the infant has thrush, he should be treated with an oral anti-fungal suspension like nystatin. This is given 4 times daily after feedings. Since the mouth of the infant is in contact with the mother’s nipples during breastfeeding, and they sound infected too, the mother and infant should be treated simultaneously. Care should be given so that the mother gently washes her nipples and dries them before breastfeeding. This will minimize or eliminate ingestion of the topical anti-fungal in the infant. Question: How should the class effect of the nasal steroids be described? There is a lot of variation among agents within the class. IncorrectThere are no significant systemic effects with these. CorrectThere are high rates of nasal bleeding.It is generally not well-tolerated. Explanation: One reason these are preferred agents in older adults (and young children) is that there are very few systemic effects. The steroids are absorbed across the mucous membranes and are deposited in the area where they need to work. There is absolutely no sedation associated with their use. They are generally well- tolerated. The class is predictable. There is very little variation between agents. Question: A 70 year-old male has a yellowish, triangular nodule on the side of the iris. This is probably: a stye.a chalazion.a pinguecula. Correctsubconjunctival hemorrhage. Explanation: Pinguecula are common as patients age. They usually appear on the nasal side first and then on the temporal side. This is a completely benign finding. A stye is also called a hordeolum. It is a tender, painful infection of a gland at the eyelid margin. These are self-limiting. A chalazion is a non-tender enlargement of a meibomian gland. A subconjunctival hemorrhage is a blood red looking area on the sclera that does not affect vision. It occurs and resolves spontaneously. Question: A patient with allergic rhinitis developed a sinus infection 10 days ago. He takes fexofenadine daily. What should be done with the fexofenadine? Stop the fexofenadine. Stop the fexofenadine and add a nasal steroid. IncorrectContinue the fexofenadine and prescribe an antibiotic. CorrectContinue the fexofenadine and add a decongestant. Explanation: This patient should continue his fexofenadine. This treats his allergies and although he has a sinus infection, he still needs treatment for his allergies. A topical nasal steroid can be added if poor control of APEA TEST BANK-with 100% verified answers-2024-2025 South University allergies exists, otherwise, this probably just increases the cost of treatment of during this sinus infection. If his sinus infection has been present for 10 days, an antibiotic seems prudent at this point. Question: A patient who is 65 years old states that she has “hayfever” and has had this since childhood. What agent could be safely used to help with rhinitis, sneezing, pruritis, and congestion? Nasal steroid CorrectIpratropiumAntihistamine IncorrectDecongestant Explanation: A nasal steroid is considered the gold standard for improvement of all symptoms associated with allergic rhinitis (hay fever). Ipratropium helps with rhinitis only. An antihistamine helps will all symptoms listed but is not as effective at relieving symptoms of congestion as a nasal steroid. A decongestant is extremely effective at relieving congestion, but no other symptoms listed. Question: A patient presents to your clinic with a painless red eye. Her vision is normal, but her sclera has a blood red area. What is this termed? ConjunctivitisAcute iritisGlaucomaSubconjunctival hemorrhage Correct Explanation: This represents leakage of blood out of the ophthalmic vasculature. It is usually painless and can be the result of coughing, sneezing, hypertension, or trauma. This will resolve without treatment, but, aspirin or other agents that can produce bleeding should be discontinued until the etiology is determined. Question: A 30 year old male has been diagnosed with non-allergic rhinitis. What finding is more likely in non- allergic rhinitis than allergic rhinitis? Older age of symptom onset CorrectMale genderPost nasal drip IncorrectSneezing Explanation: Non-allergic rhinitis, often called vasomotor rhinitis, is very common in the US. It is typically diagnosed and differentiated from allergic rhinitis by history. Although both conditions may co-exist in patients, non- allergic rhinitis typically has onset after age 20 years. Allergic rhinitis typically presents prior to age 20 years. The most common symptoms associated with non-allergenic rhinitis are nasal congestion and post- nasal drip. It is predominantly reported in females. Common precipitants of non-allergic rhinitis symptoms can occur with exposure to spicy foods, cigarette smoke, strong odors, perfumes, and alcohol consumption. This is frequently treated with topical azelastine. Question: An elderly patient who has a red eye with tearing was diagnosed with conjunctivitis. What characteristics below indicate viral conjunctivitis? Moderate tearingProfuse tearing CorrectModerate exudate IncorrectProfuse exudate Explanation: A patient with viral conjunctivitis typically has profuse tearing and minimal exudate. Patients with bacterial conjunctivitis have moderate tearing and exudate. Both are considered highly contagious. Patients should refrain from work or group activities for 48 hours until symptoms have subsided. Question: An NP examines a screaming 2 year-old. A common finding is: APEA TEST BANK-with 100% verified answers-2024-2025 South University nasal discharge.increased respiratory rate. Incorrectpink tympanic membranes. Correctcoarse breath sounds. Explanation: The tympanic membrane normally becomes pink and can rarely become red when a child is screaming or crying. This is probably due to flushing and hyperemia of the face that occurs with crying. A distorted or erythematous tympanic membrane with decreased mobility is suggestive of otitis media. Question: When examining the vessels of the eye: the veins are smaller than the arteries.the arteries are smaller than the veins. Correctthe arteries are dark red. Incorrectthe arteries pulsate. Explanation: The arteries are 2/3 to 4/5 the diameter of the veins. The arteries appear as light red in color; veins are darker red. Interestingly, the veins in the eyes pulsate; the arteries do not. Loss of venous pulsations can be identified in patients with head trauma, meningitis, or elevated intracranial pressure. Question: A patient presents with severe toothache. She reports sensitivity to heat and cold. There is visible pus around the painful area. What is this termed? Pulpitis CorrectCariesGingivitisPeriodontitis Incorrect Explanation: The predominant symptom of patients who exhibit pulpitis is pain especially elicited by thermal changes, cold and hot. The pain can become severe and patients are ill appearing. Pus may be seen around the gum area or may be restricted to the pulp cavity. Caries and gingivitis do not produce pus. Periodontitis is characterized by gingival inflammation and pain. Pus is not present in this disease. A periodontal abscess produces pain and pus, but the pus is usually only expressed after probing. Question: What medication should always be avoided in patients with mononucleosis? ClindamycinIbuprofen IncorrectAmoxicillin CorrectTopical lidocaine Explanation: A generalized rash may be seen in patients with mononucleosis (mono) who are given amoxicillin or ampicillin at the time of the acute phase of the illness. The rash does not represent an allergic reaction, but instead probably represents a reaction between the Epstein Barr virus and the penicillin molecule. The rash is usually described as maculopapular and may be pruritic. The rash has also been described with other beta-lactam antibiotics, azithromycin, cephalexin and levofloxacin. Question: A 3 year-old has been diagnosed with acute otitis media. She is penicillin allergic (Type 1 hypersensitivity reaction). How should she be managed? Amoxicillin Amoxicillin-clavulanateCefuroxilClarithromycin Correct Explanation: This patient experienced a Type I allergic reaction to penicillin. This is characterized by hives, wheezing, or anaphylaxis. It is NEVER considered safe to prescribe a cephalosporin. Macrolides may be prescribed to patients with a true Type 1 reaction to penicillin. Since amoxicillin is a penicillin, it should not be prescribed. APEA TEST BANK-with 100% verified answers-2024-2025 South University Explanation: The “get up and go” test is used to evaluate musculoskeletal function. The patient is asked to rise from a seated position in an armchair, walk across the room, turn around, and return to the chair. This test evaluates the patient’s gait, balance, leg strength, and vestibular function. It should be assessed in patients who report a fall or who present after a fall but who appear without injury. Question: A patient complains of right leg numbness and tingling following a back injury. He has a diminished right patellar reflex and his symptoms are progressing to both legs. What test should be performed? Lumbar x-rays IncorrectLumbar CT scanLumbar MRI CorrectLumbar MRI with contrast Explanation: This patient has symptoms that could indicate an urgent neurological situation. Acute radiculopathy could indicate the need for intervention by a neurosurgeon. An MRI is a superior study because it provides excellent information about the soft tissues, like the lumbar discs. Contrast might be used in this patient if he had a history of previous back surgery. Then, contrast would be helpful to distinguish scar tissue from discs. Question: An elderly patient is at increased risk of stroke and takes an aspirin daily. Aspirin use in this patient is an example of: primary prevention. Correctsecondary prevention.tertiary prevention. Incorrectprimary or secondary. Explanation: Primary prevention refers to an action that has the potential to prevent an event prior to its occurrence. Secondary prevention refers to an intervention demonstrated to help prevent a second occurrence of a deleterious event; or may refer to an intervention designed for early detection. Tertiary prevention is an action designed to prevent additional deleterious events from occurring. Question: A patient presents to the NP clinic with a complaint of nocturnal paresthesias. What is the likely underlying etiology? De Quervain’s tenosynovitisCarpal tunnel syndrome CorrectUlnar radiculopathy IncorrectMedial epicondylitis Explanation: Nocturnal paresthesias are typical in patients who have carpal tunnel syndrome. A patient will complain of nighttime numbness, tingling, or “sleeping” hands and arms. This is a result of compression of the median nerve that traverses through the carpal tunnel. If the nerve is compressed, the symptoms (nocturnal paresthesias) usually result. With surgical decompression, symptoms usually abate. Question: Mrs. Jopson is unable to name a familiar object. How is this described? Anomia CorrectAnosmiaAcanthosisIncompetent Explanation: Anomia is the difficulty in the naming of familiar objects. This is an example of mild impairment. Other evidence of mild impairment is recent recall problems, decreased insight, and difficulty managing finances. Many mildly impaired adults are not able to state today’s date. APEA TEST BANK-with 100% verified answers-2024-2025 South University Question: Mrs. Johnson is an 89 year-old resident at a long-term care facility. Her state of health has declined rapidly over the past 2 months, and she can no longer make her own decisions. Her daughter requests a family conference with the nurse practitioner. Some important principles that need discussion at this time, if not previously documented, are: bereavement support for the family, quality of life for the resident, and living will. Explanation: American Geriatrics Society stresses not only care of the patient, but care of the family as well. Choice A includes meeting the current and future needs of the patient, family needs, and end of life issues with the living will. The living will is recognized as a valid advanced directive. Choice A includes developmental landmarks for the patient and family. Question: A patient diagnosed with cluster headaches: usually has scotomas.can be diagnosed with an imaging study.should eliminate triggers like nicotine and alcohol. Correctmay exhibit nuchal rigidity. Explanation: Cluster headaches are extremely painful headaches but are not as common as migraine or tension headaches. The headaches occur in cyclical patterns, hence the name “cluster”. The cycle lasts about 2-12 weeks. A single attack may last 30-90 minutes, but can last up to 3 hours. Patients do not exhibit fever or nuchal rigidity. These symptoms may characterize meningitis. Lifestyle measures like avoiding alcohol, nicotine, and high altitudes may help prevent a headache . Avoiding afternoon naps, bright lights, and glare during a headache cycle, may prevent a subsequent headache from occurring. Scotoma refers to an area of diminished vision in the visual field. Question: A patient who had a stroke has recovered and is performing all of her activities of daily living. Taking aspirin for stroke prevention is an example of: primary prevention.secondary prevention. Correcttertiary prevention. Incorrectquaternary prevention. Explanation: The patient is taking aspirin to prevent recurrent stroke. Research demonstrates that taking an aspirin daily can prevent subsequent strokes and MI significantly. Secondary prevention means that the intervention is performed to prevent another occurrence of the deleterious event. If she had never had a stroke but took an aspirin daily for prevention of stroke, that would be primary prevention. Taking an aspirin at home during the course of having an MI is an example of tertiary prevention. There is no reference in the literature to quaternary prevention. Question: An elderly patient with organic brain syndrome is at increased risk of elder abuse because she: lives in a nursing home.has multiple caregivers.is incontinent of stool and urine.has declining cognitive function. Correct Explanation: Elders are at increased risk of abuse because of their decline in cognitive function. Caregiver strain, stress, and depression occur at higher rates than in the general population. According to the National Center of Elder Abuse, family members are likely to be abusers of the infirmed elderly. Healthcare providers should remain alert to signs of elder abuse and caregiver stress. Question: When should medications be started in a patient diagnosed with Parkinson’s disease? As soon as the disease is diagnosedWhen symptoms interfere with life’s activities CorrectWhen non- pharmacologic measures have been exhaustedAfter MRI and CT have ruled out stroke or tumor APEA TEST BANK-with 100% verified answers-2024-2025 South University Explanation: The medications used to treat patients with Parkinson’s disease do not prevent progression of the disease. Therefore, it is not necessary to start them until the patient’s symptoms dictate that he desires more control of movements. The dose of medications is always the lowest one that achieves control of symptoms. It can be titrated upward as needed to control symptoms. Orthostatic hypotension is a common side effect of the medication and so should be monitored closely. Question: The Snellen chart is used to assess: distant vision Question: A 26 year-old patient with long history of chronic sinusitis presents today with temperature of 103.2 F, headache, and stiff neck. Which finding below should make the nurse practitioner suspect meningitis? Positive Kernig’s and Brudzinski’s signs CorrectHistory of HIVPhotophobiaDecreased cervical range of motion Incorrect Explanation: These two tests demonstrate nuchal rigidity. Brudzinski’s sign refers to spontaneous flexion of the hips during attempted passive flexion of the neck. Kernig’s sign refers to the inability to allow full extension of the knee when the hip is flexed 90 degrees. In addition to the finding of nuchal rigidity, headaches, fever, and altered mental status characterize acute, bacterial meningitis. Question: A patient is diagnosed with carpal tunnel syndrome. Which finger is not affected by carpal tunnel syndrome? Fifth finger Question: A 62 year-old female patient presents to the clinic with very recent onset of intermittent but severe facial pain over the right cheek. She is diagnosed with trigeminal neuralgia. What assessment finding is typical of this? Pain is much worse with sticking her tongue out IncorrectPain is better with light touch over the affected areaPain is relieved with NSAIDsPain may be triggered with touch of the right cheek Correct Explanation: Trigeminal neuralgia is a common cause of severe facial pain. It is described by patients as electric or shocking pain. Triggers for pain are light touch to the affected area, chewing, cool breeze on the cheek, and smiling or grimacing. It is more common in older adults. Question: In a three year-old with fever, which finding might precipitate a febrile seizure? A sudden decrease in body temperature after high fever CorrectA worsening pneumonia IncorrectInitiation of an antibioticReturning to daycare prior to being fever free for 24 hours Explanation: A sudden rise or fall in body temperature lowers the seizure threshold in children as well as adults. APEA TEST BANK-with 100% verified answers-2024-2025 South University Explanation: This patient likely suffered a transient ischemic attack. He needs urgent evaluation with head CT and/or MRI, ECG, lab work (CBC, PTT, lytes, creatinine, glucose, lipids and sedimentation rate); possible magnetic resonance angiography, carotid ultrasound, and/or transcranial Doppler ultrasonography. He is at increased risk of stroke within the first 48 hours after an event like this one. On initial evaluation, the most important determination to be made is whether the etiology of the stroke or TIA is ischemic or hemorrhagic. After this determination, treatment can begin. Unfortunately, this determination cannot be made in the clinic. The patient needs urgent referral to a center where this evaluation and possible treatment can be performed. Question: A 72 year-old patient with a relatively benign medical history complains of new onset headache associated with abrupt onset of visual disturbances. Her sedimentation rate is elevated. Her neuro exam is otherwise normal. What is the most likely reason for her symptoms? StrokeMigraine headache IncorrectBrain tumorTemporal arteritis Correct Explanation: Giant cell arteritis, also known as temporal arteritis, is most likely in this patient based on the results of the exam and lab. This is best diagnosed by temporal artery biopsy. Temporal arteritis is a chronic vasculitis of the medium and large vessels. A biopsy can be performed on the same day as it is suspected. Generally, patients have complaints of new onset headaches, abrupt onset of visual disturbances, jaw claudication, unexplained fever or anemia, and an elevated sedimentation rate. The average age of diagnosis is 72 years. She should be referred to neurology for evaluation today. Question: A nurse practitioner is assessing a 3 day old infant’s head. What would be a normal observation in a healthy 3 day old infant who is crying? There are pounding pulsations over the anterior fontanel.There are palpable pulsations over the anterior fontanel. CorrectThere is a palpable nodule near the posterior fontanel.The fontanel is depressed. Explanation: Palpable or visible pulsations are common if the infant is crying or is agitated. Normally, the anterior fontanel is slightly depressed, but a depressed fontanel is indicative of a dehydrated or malnourished infant. Pounding pulsations are indicative of increased intracranial pressure. A palpable nodule near the posterior fontanel is an abnormal finding possibly indicative of a calcium deposition or bony abnormality. Question: What is the earliest age that an average child would appropriately receive construction paper and a pair of scissors with rounded points? 3 years4 years Correct5 years6 years Incorrect Explanation: A 4 year-old should have the dexterity to cut and paste. Therefore, this is the most appropriate age. A 5 or 6 year-old may also use these items, but these would be appropriate as early as 4 years. Pointed tips represent hazards especially in a classroom with multiple children moving simultaneously. Question: Which reflex may be present at 9 months of age during sleep? Moro reflexRooting CorrectStepping IncorrectTonic neck APEA TEST BANK-with 100% verified answers-2024-2025 South University Explanation: The rooting reflex disappears about 2-3 months of age but may be present during sleep up to 12 months. The Moro reflex disappears somewhere between 3 and 6 months, usually 4-5 months. The tonic neck, or “fencing” response, generally disappears about 6 months of age. The stepping response disappears about 9 months. Interestingly, this is usually when infants are beginning to take the first steps with assistance. Question: A 68 year-old smoker with a history of well controlled hypertension, describes a syncopal episode which occurred yesterday while mowing his lawn. Today, he has no complaints. Initially, the NP should: perform a complete neurological and cardiac exam with auscultation of the carotid arteries. Correctorder a 12 lead ECG, carotid ultrasound, and perform a physical exam.order a CT of the brain, blood clotting studies, and cardiac enzymes.check blood pressure in 3 positions, order a 12 lead ECG, and schedule an exercise stress test. Incorrect Explanation: The event described is syncope. Syncope is a brief and sudden loss of consciousness that occurs with spontaneous recovery. This is a significant event but especially in a smoker with hypertension. The initial assessment of this patient must start with an examination of the patient’s cardiac and neurological systems. Based on the findings and tentative diagnosis of syncope, coupled with the patient’s history of the event, other tests might be ordered to evaluate arrhythmias, stroke, transient ischemic attack, myocardial infarct, carotid stenosis, etc. A referral to specialty care is indicated after initial workup by the nurse practitioner. Question: The most common polyneuropathy in the elderly is: Charcot-Marie-Tooth disease.diabetes mellitus. Correcturinary incontinence.Guillain-Barre syndrome. Explanation: A polyneuropathy is a term that refers to a process that affects multiple nerves, usually peripheral. The distal nerves are more commonly affected. Symptoms described by patients are burning, weakness, or loss of sensation. Charcot-Marie-Tooth disease is a rare, hereditary primary motor sensory neuropathy. Guillain- Barre is an acute autoimmune neuropathy that is primarily demyelinating. Urinary incontinence does not represent a common polyneuropathy in the elderly. Question: Restless legs syndrome is part of the differential diagnosis for Mr. Wheaton. What should be part of the laboratory workup? BUN/CrSerum ferritin CorrectALT/AST IncorrectUrinalysis Explanation: Restless legs syndrome (RLS) is the unrelenting urge to move the legs. This rarely affects the upper extremities. The symptoms are relieved by movement of the affected limbs and only occurs if the affected limbs are at rest. Iron deficiency has been considered as a cause of RLS. The exact mechanism of iron deficiency is not known, but many patients who exhibit symptoms of restless legs syndrome have low serum ferritin levels and have relief of symptoms when supplemented with iron. Even in patients with normal serum levels, a month long trial of iron may be helpful. Question: A 70 year-old male who is diabetic presents with gait difficulty, cognitive disturbance, and urinary incontinence. What is part of the nurse practitioner’s differential diagnosis? Diabetic neuropathyNormal pressure hydrocephalus CorrectParkinson’s DiseaseMultiple APEA TEST BANK-with 100% verified answers-2024-2025 South University sclerosis Incorrect APEA TEST BANK-with 100% verified answers-2024-2025 South University Explanation: This scenario is typical of an infant with pyloric stenosis. It is more common in males (88%) and usually is diagnosed before the child is 12 weeks old. The classic presentation is an infant who vomits immediately after eating. The “knot” in the abdomen is the typical “olive-shaped mass” palpable at the lateral edge of the rectus abdominus muscle in the right upper quadrant of the abdomen. The mass is best palpated immediately after vomiting. The differential should include GER, but no mass is palpable. Munchausen syndrome by proxy is the fabrication or induction of an illness in a child in order for the caregiver to receive attention. Question: A mother of a 4 week-old infant visits your office. She states that her baby is vomiting after feeding and then cries as if he is hungry again. What should the nurse practitioner assess? His abdomen for an olive shaped mass CorrectHis rectum for patencyHis swallowing abilityThe position his mother uses when she feeds him Explanation: The symptoms sound like pyloric stenosis. The most common time for this to occur is 3-6 weeks. It rarely occurs after 12 weeks of age. Babies who have episodes of projectile vomiting and who demand to be re- fed are called “hungry vomiters” and a diagnosis of pyloric stenosis should be considered. The olive shaped mass, if found, probably represents hypertrophy of the lateral edge of the rectus abdominus muscle. It is most easily felt immediately after vomiting. This diagnosis is made much earlier now than it used to be, so problems with dehydration are not as common as decades ago. Question: The relationship between colon polyps and colon cancer is: polyps eventually all become malignant.polyps have a slow progression to colon cancer. Correctpolyps have a rapid progression to colon cancer.polyps have no relationship to colon cancer. Explanation: Colon polyps are usually very slow growing and take a long time to progress to cancer. This is the reason that a colonoscopy does not need to be repeated annually. While not all polyps grow slowly, this is the usual progression. Question: Which of the following is appropriate for initiation of an 8 week-old with gastroesophageal reflux? Small, frequent thickened feedings CorrectCimetidine every 6 hoursChange formula to soy based IncorrectPlace infant on left side after eating Explanation: Two strategies should be tried initially. First, avoidance of overfeeding is recommended. Hence, small, frequent feedings. Second, milk thickening agents appear to improve symptoms in infants who experience gastroesophageal reflux (GER). Thickened feedings significantly decrease frequency of reflux in most infants. Also, caloric content is increased and this may be helpful for patients who are underweight because of persistent GER. Generally, when medications are used, proton pump inhibitors are preferred over an H2 blocker like cimetidine. Changing formula generally does not help, however, a milk-free diet may help since 40% of infants with GER are sensitive to cow’s milk protein. Thus, soy would not help. Positioning seems to be ineffective in relieving symptoms in infants. APEA TEST BANK-with 100% verified answers-2024-2025 South University Question: A patient has a positive hepatitis B surface antibody. This means: he has acute hepatitis B.he has chronic hepatitis B.he is immune to hepatitis B. Correcthe needs immunization to hepatitis B. Explanation: The hepatitis B surface antibody indicates immunity to hepatitis B virus. Specifically, if this patient comes in contact with hepatitis B virus, he will not become infected with hepatitis B. The presence of hepatitis B surface antibody indicates immunity from immunization or actual infection. It also indicates recovery if the patient was infected. Question: An elderly adult has chronic constipation. How should this be managed initially? Avoid all constipating medications/foods when possible CorrectAdd dietary fiber and increase fluidsAdd sorbitol solution dailyUse an oil retention enema Explanation: Initially, all medications known to cause constipation should be stopped when possible. Symptoms should be re-assessed. It is always preferable to stop a medication to correct a condition BEFORE adding a medication to correct a condition. If symptoms persist, 6-25 grams of dietary fiber should be added along with increasing fluid intake. Physical activity should be increased as tolerated. Choice 3 and 4 would be initiated if other options mentioned were not helpful. Question: GERD (gastroesophageal reflux disease) and physiologic reflux have similar characteristics. However, physiologic reflux: can produce mucosal injury. Incorrectrarely occurs at nighttime. Correctoccurs only postprandial.is always asymptomatic. Explanation: GERD is usually associated with symptoms that produce injury to the mucosa. This rarely occurs in physiologic reflux. Characteristics typical of physiologic reflux are that it occurs primarily post-prandial (like GERD), are usually asymptomatic, rarely occur nocturnally, and are usually short-lived. In contrast, GERD usually occurs nocturnally, especially when there is lower esophageal sphincter disease. Question: Children with an inguinal hernia: have a history of an intermittent bulge in the groin. Correctare usually symptomatic.have a mass that is always present on exam.are usually irritable and often constipated. Incorrect Explanation: Inguinal hernias are usually asymptomatic and absent on exam but can sometimes be elicited by increasing intraabdominal pressure such as occurs with straining or crying. The “silk sign” is infrequently appreciated but represents a silky thickening of the cord. If it is able to be palpated, it is done by placing a single finger next to the inguinal canal at the level of the pubic tubercle and gently moving the finger from side to side. Children with an incarcerated mass are often irritable but not constipated. Question: Which of the following would be usual in a patient with biliary colic? APEA TEST BANK-with 100% verified answers-2024-2025 South University Presence of gallstones on imaging studies Presence of gallstones and unpredictable abdominal painPositive Murphy’s sign only Pain in upper abdomen in response to eating fatty foods Correct Explanation: Biliary colic refers to discomfort produced by contraction of the gallbladder. This occurs in response to eating. Typically, pain occurs in the upper right quadrant or chest, peaks in an hour after eating, and then remains constant and finally subsides over the next several hours. A positive Murphy’s sign is elicited when the gallbladder wall is inflamed. It can be elicited by palpating the gallbladder just beneath the liver as the patient takes a deep breath. Question: A 40 year-old patient has the following laboratory values. How should they be interpreted? HBsAg (-), HBsAb (+), HBcAb (-) The patient had hepatitis.The patient has hepatitis. IncorrectThe patient should consider immunization.The patient has been immunized. Correct Explanation: This patient has a negative hepatitis B surface antigen (HBsAg). Therefore, he does not have hepatitis B. The patient has a negative hepatitis B core antibody (HBcAb). Therefore, he has never had hepatitis B. The patient has a positive hepatitis B surface antibody (HBsAb). Therefore, he is considered immune. The patient is immune from immunization because his hepatitis B core antibody is negative. If the core antibody had been positive, he would be considered immune from the disease. The correct answer is choice D. Question: A 95 year-old male has lost muscle mass as he has aged. He does not have any underlying disease that has caused this loss. What is this termed? HypoproteinemiaSarcopenia CorrectCachexia IncorrectDithering Explanation: Sarcopenia is the term used to describe loss of muscle mass related to aging. The actual definition is a decrease in appendicular muscle mass that is measured as a two standard deviation decrease. It is associated with increased falls, disability, and impairment. In males, this is due to decreased production of testosterone. Question: Hirschsprung's disease is characterized by: intermittent constipation in the first year of life.inability to absorb carbohydrates.failure to pass meconium in the first 48-hours of life. Correctchronic fecal incontinence. Explanation: Another name for this condition is aganglionic megacolon. The affected segment of colon is absent of ganglia that are responsible for producing peristalsis. The diagnosis cannot be made in the first 48-hours of life, but a suggestion of the diagnosis can be made because meconium is not passed in a timely manner. Consequently, most children are diagnosed in the neonatal period. Patients usually present with emesis, abdominal distention, and failure to pass stool. An abdominal x-ray will demonstrate distal intestinal obstruction with dilated bowel loops. The patient should be referred for surgery to remove the diseased segment of colon. APEA TEST BANK-with 100% verified answers-2024-2025 South University symptoms in older adults can manifest as lower abdominal pain and is a common presentation in this age group. Low grade fever is common too. Question: A patient with gall bladder disease has classic symptoms. Which symptom below is NOT classic of gallbladder disease? Intense, dull pressure in the mid abdomenPain that radiates into the chest, back, or right shoulder bladePain that worsens after a fatty mealPain that occurs when the stomach empties Correct Explanation: The pain associated with cholecystitis is usually constant and may or may not occur in relation to meals, but, it rarely occurs after the stomach has emptied (takes several hours) or with fasting. Initial pain is usual after a fatty meal. The pain is usually in the upper right quadrant but is nearly as common in the mid abdomen. The pain can mimic a myocardial infarction and so it must be treated as cardiac pain until proven otherwise. The pain is usually caused by contraction of the gallbladder after a meal. The contraction may force a stone or sludge against the gallbladder outlet. Discomfort can last for several hours. Question: A fecal occult blood test (FOBT) obtained during a rectal examination: should be adequate for screening of colorectal cancer.will usually detect the presence of polyps.is adequate to screen for rectal cancer only.is inadequate to screen for colorectal cancer. Correct Explanation: This is not adequate to screen for colorectal cancer. Fecal occult blood tests have a much higher sensitivity if three consecutive stool specimens are used (applying 2 samples per card for each specimen). A single specimen is inadequate for screening purposes. Since polyps do not usually bleed, the fecal occult blood test is not a good screen for polyp identification. Question: Which description is more typical of a patient with acute cholecystitis? The patient rolls from side to side on the exam table.The patient is ill appearing and febrile. CorrectAn elderly patient is more likely to exhibit Murphy’s sign.Most are asymptomatic until a stone blocks the bile duct. Explanation: A patient with acute cholecystitis usually complains of abdominal pain in the upper right quadrant or epigastric area. Many patients complain of nausea. The patient lies very still on the exam table because cholecystitis is associated with peritoneal inflammation that is worse with movement. Elderly patients are more likely to NOT exhibit Murphy’s sign and thus, are more likely to suffer from complications of acute cholecystitis. Patients who are asymptomatic have cholelithiasis, not acute cholecystitis. Question: A 14 year-old patient has an acute, painless groin swelling. What tool would yield the most information to identify the etiology of the swelling? Abdominal radiographsUltrasound of the abdomenUltrasound of the scrotum CorrectMRI of the scrotum Explanation: The patient has an acute swelling of the groin. Since the etiology could include several scrotal problems (inguinal hernia, hydrocele, or varicocele), an ultrasound will yield quick reliable information with a APEA TEST BANK-with 100% verified answers-2024-2025 South University diagnostic accuracy of 93% for acute groin problems. The definitive treatment for inguinal hernia is surgical repair. Question: The most common place for indirect inguinal hernias to develop is: the internal inguinal ring. Correctthe external inguinal ring.Hesselbach’s triangle.femoral ring. Incorrect Explanation: The internal inguinal ring is the most common site for development of an indirect inguinal hernia. These can occur in men and women. Though most are probably congenital, symptoms may not be obvious until later in life. Indirect hernias are more common on the right side. Direct inguinal hernias occur through Hesselbach’s triangle. Question: A patient has a positive anti-HCV test. This means: he has hepatitis C. Incorrecthe has immunity to hepatitis C.he does not have hepatitis C.more data is needed. Correct Explanation: The anti-HCV test is a screening test. A patient who has a positive screen for hepatitis C may have the disease, but more data is needed to determine this. Once the screen is positive, a confirmatory test should be performed. The confirmatory test is the RIBA (recombinant immunoblot assay). If it is positive, it indicates past or current infection with hepatitis C. If negative, the RIBA indicates that the screen was a false positive. Question: A 5 year-old has been diagnosed with pinworms. He lives with his mother. There are no other members of the household. How should his mother be managed? Reassure the mother that if she develops symptoms, she will need to be treated.Visually assess the mother’s rectum for redness or presence of worms.Have the mother collect a stool specimen and send it to the laboratory. IncorrectPerform the “scotch tape” test and look at the collection under the microscope. Correct Explanation: The diagnosis of pinworms (Enterobiasis) is made by using a piece of scotch tape on a tongue depressor. It is touched against the patient’s rectum. The greatest yield of eggs will occur during the nighttime or early AM. Eggs will be found here if they are present. Worms and eggs are rarely found in stool specimens, so this is not a good plan. When the scotch tape is examined under a low power microscope, the eggs will be easily visualized since they are large and bean shaped. The finding of an adult worm would confirm the diagnosis. These are large enough to be seen with the naked eye. If the mother is symptomatic, she should be treated with or without a rectal exam. It is very likely she is infected. Question: Older adults frequently complain of constipation. Which medication listed below does NOT increase chance of constipation in an older adult? Diltiazem IncorrectHydrochlorothiazideCalcium supplementsMetformin Correct APEA TEST BANK-with 100% verified answers-2024-2025 South University Explanation: Constipation is a common complaint in older adults because of mediations they routinely consume and many diseases they often have. Some other examples of medications that can cause constipation are opiate analgesics, NSAIDs, and antacids. Some diseases and conditions that can produce constipation are hypothyroidism, colon cancer, electrolyte abnormalities, immobility, change in diet due to inability to chew food properly. Question: What medication used to treat patients who have GERD provides the fastest relief of heartburn symptoms? Calcium carbonate CorrectRanitidine IncorrectAmantadinePantoprazole Explanation: Calcium carbonate is an antacid. It provides rapid changes in gastric pH. This provides relief that can be noticed immediately. The increase in pH lasts for about 30 minutes. Ranitidine is an H2 blocker. It provides relief in 1-2 hours. This usually lasts for about 6-12 hours. Amantadine is an antiviral not used to treat GERD. Pantoprazole is a proton pump inhibitor. This provides relief after several hours or days of daily consumption. Question: The most common cause of diarrhea in adults is: E. coli.salmonella.C. difficile.viral gastroenteritis. Correct Explanation: Most cases of acute gastroenteritis are viral in origin. Severe diarrhea is usually caused by bacteria. This typically lasts longer than 3 days. Question: A patient has hepatitis B. He probably has a predominance of: leukocytes.lymphocytes. Correctneutrophils.eosinophils. Explanation: Lymphocytes tend to be the predominant white cell present during viral infections. Hepatitis B is a viral infection. The total white count will likely be decreased. This happens very often in the presence of viral infections. A bacterial infection is frequently evidenced by an elevated leukocyte count, a increased neutrophil count, and a decreased lymphocyte count. Question: A 19 year-old female presents with lower abdominal pain that began about 12 hours ago. She is febrile. She denies vaginal discharge. Which choice below is the least likely cause of her symptoms? AppendicitisUrinary tract infectionRenal stone CorrectOvarian cyst Incorrect Explanation: A renal stone can produce lower abdominal pain, but is unlikely to produce fever. Fever can be associated with the presence of a stone if pyelonephritis also accompanies this. However, this is not nearly as likely as the other diagnoses listed. Another diagnosis that must be considered because of her age, is pelvic inflammatory disease. In this case, a pelvic exam should be performed. APEA TEST BANK-with 100% verified answers-2024-2025 South University can produce pancreatitis. APEA TEST BANK-with 100% verified answers-2024-2025 South University Question: What medication listed below could be used to increase appetite in an anorexic patient? Megestrol CorrectLoratadineFexofenadineSerum ferritin Explanation: Megestrol acetate has a positive effect on weight in patients who are trying to gain weight. It increases appetite and may improve quality of life. Weight gain may take up to 3 months before it is measurable. Megestrol has the potential to cause edema and so should be used cautiously (or not at all) in patients with chronic heart failure. Question: A 15 year-old is about 10% below her ideal body weight. She complains of dizziness when she stands up. Laboratory studies were performed. Besides malnutrition, what else could account for her dizziness? BUN is mildly elevatedGlucose = 80 mg/dLHemoglobin = 9.6 mg/dL CorrectPotassium is 3.5 meq/L Explanation: This patient does not meet the strict criteria for an eating disorder, but it should be suspected. The other laboratory values are not the cause of her dizziness with standing. This is likely due to a low hemoglobin. She needs treatment for a probable iron deficiency anemia and elicitation of history to help identify the cause of her low hemoglobin. Question: A 40 year-old patient has the following laboratory values. How should they be interpreted? HBsAg (-), HBsAb (+), HBcAb (+) The patient had hepatitis. CorrectThe patient has never had hepatitis.The patient should consider immunization.The patient has been immunized. Incorrect Explanation: This patient has a negative hepatitis B surface antigen (HBsAg). Therefore, he does not have hepatitis B. The patient has a positive hepatitis B surface antibody (HBsAb). Therefore, he is considered immune. The patient also has a positive hepatitis B core antibody (HBcAb). Therefore, he is immune because he has had hepatitis B. The correct answer is choice A. Question: What is true regarding overweight states in older adults? This is clearly associated with increased mortality in older adults. IncorrectMortality in the elderly related to overweight states declines over time. CorrectBMI is a good way to assess nutritional states in the elderly.There are no potential metabolic or functional benefits to weight loss in the elder. Explanation: Overweight and obese states are not as important in predicting mortality in the elderly as they are in their younger counterparts. After age 65 years (some studies demonstrate after age 70), weight is less significant in decreasing risk for mortality than in younger adults. There are some benefits to weight loss in the obese elderly. One of them is better balance and decreased risk for falls. Others include less sleep apnea, decreased risk of diabetes, decreased rates of shortness of breath with respiratory and cardiac diseases. APEA TEST BANK-with 100% verified answers-2024-2025 South University Question: A 45 year-old patient has the following laboratory values. How should they be interpreted? HBsAg (+), HBsAb (-), HBcAb (+) The patient has hepatitis. CorrectThe patient had hepatitis. IncorrectThe patient should consider immunization.The results are indeterminate. Explanation: This patient has a positive hepatitis B surface antigen (HBsAg). Therefore, he has hepatitis B. A positive HBcAb is found in patients who either have hepatitis now or who have had it. The surface antibody (HBsAb) would be expected to be negative in a patient with positive surface antigen (HBsAg) because these 2 markers will not be positive at the same time. Question: Which medication listed below can exacerbate the symptoms of GERD? Verapamil CorrectMetforminFerrous sulfateCeftriaxone Explanation: Verapamil is a calcium channel blocker. Calcium is needed for muscle contraction. Since the lower esophageal sphincter is opened and closed by muscles, the contraction of these muscles will be less forceful. GERD can be exacerbated in this case. Calcium channel blockers should be avoided in patients with severe GERD or in patients in whom calcium channel blockers exacerbate GERD symptoms. Question: A patient has been prescribed metronidazole for treatment of C. difficile. What should be avoided in this patient? Excess fluidsVitamin B12Grapefruit juiceAlcohol Correct Question: A 50 year-old with a history of consumption of 3-4 alcoholic drinks daily and weekend binges has elevated liver enzymes. Which set of enzymes is most representative of this patient? AST= 200, ALT= 75 CorrectAST= 100, ALT = 90AST=100, ALT=200AST= 30, ALT= 300 Explanation: The normal AST/ALT ratio in healthy subjects is 0.8. In patients with alcoholic hepatitis, the usual ratio (AST:ALT) is 2:1. When the ALT is very elevated, infectious hepatitis must be considered. Normally, both AST and ALT are less than 40 IU/L. The level of elevation does not correlate with the degree of damage in the liver and has no prognostic value in patients with non-acute liver disease. Question: The most common risk factor for developing hepatitis B is: homosexual activity.injecting drug use. Incorrectheterosexual activity. Correctbody piercings.